Qureshi University, Advanced courses, via cutting edge technology, News, Breaking News | Latest News And Media | Current News
admin@qureshiuniversity.com

Admissions | Contact Us | Examinations | Grants | Instructors | Lecture | Membership | Recommendations | Research Grants | Librarians | Booksellers | Forms | Continents/States/Districts | Contracts | Students login | Universities | Volunteer

Qureshi Medical College International
Microbiology
What is microbiology?
What are microrganisms?
What is the immunology?
What are the branches of medical microbiology?
Where do microbiologists work?
Health care, food industry, water filtration plants.
What resources do microbiologists need?
Who else studies microbiology?
Medical doctors, administrators, dentists, pharmacologists.
What is the difference between bacteria and viruses?
What is the difference between bacteria, viruses, parasites, and fungi?
What recent discoveries have been made in medical microbiology?
Medical Microbiology
What is microbiology?

Microbiology is the study of microorganisms - specifically, disease-causing microorganisms. Microbiology is responsible for identifying infectious agents in blood, urine, sputum, feces, cerebrospinal fluid, and other body fluids. The infectious agents are then tested for sensitivity to certain antibiotics used to treat infections.

Microbiology uses various techniques to identify microorganisms, including the following:

* chemical, immunological, and genetic tests
* examination under a microscope
* staining

Microorganisms can include the following:

* bacteria
* fungi
* parasites
* viruses

Common microbiology tests: Tests Uses Blood culture Can diagnose bacterial infections of the blood Culture and sensitivity of burns and wounds Can identify disease-causing organisms and test for effectiveness of antibiotics Sputum culture Can identify lower respiratory tract infections such as pneumonia Stool culture Can identify parasites that cause disease, such as pinworms Urine culture Can identify disease-causing organisms in the kidneys and urinary tract

Microbiology is the study of micro-organisms which include bacteria, viruses, protozoa, algae and fungi. These organisms are extremely important in our everyday lives. Some are responsible for a significant proportion of the diseases affecting not only humankind but also plants and animals; others are vitally important in the maintenance and modification of our environment; still others play an essential role in industry, where their unique properties have been harnessed in the production of food, beverages and antibiotics. More recently, scientists have learnt how to exploit micro-organisms in the field of molecular biology, a science which is making an enormous impact both industrially and medically. Microbiology also encompasses the study of the body's ability to mount defences against infectious microbes.

Microbes are microscopic organisms, which range in size from the lower fungi and algae through bacteria to the smallest organisms of all, the viruses. Microbiology is the study of the properties of these organisms and the roles they play in health and disease, industrial processes as well as in the environment.

Microbiology in addition to the study of microbes also encompasses a wide range of research fields; from immunology, which sets out to study how the body’s immune system protects the host from infectious disease, to microbial genetics, including the recently-developed field of recombinant DNA. Knowledge gained from the study of how bacteria carrying specially engineered DNA can economically produce new enzymes, pharmaceuticals and industrial processes is already revolutionising biotechnology industries. Microbiologists also study how to diagnose, prevent and treatinfectious diseases—important skills in this era of emerging diseases and antibiotic resistance.

Microbiology can be applied in fields as diverse as medicine and brewing, food spoilage and space science and in this regard, a knowledge of microbiology is essential for understanding and causing problems arising from pollution of the environment by detergents, insecticides, hydrocarbons, etc. Novel sources of food and new processes in mining and other industries make use of bacteria.

A knowledge of microbiology is thus useful for people pursuing careers in health, engineering, food science and the environment and the human environment as well as for those intending to pursue careers in microbiology.

Microbiology is the scientific field that is occupied with the study of microscopic organisms, commonly known as microrganisms.

What are microrganisms?

All living creatures consist of cells. Cells are very small basic units of life. They are the smallest structures capable of basic life processes, such as taking in nutrients and expelling waste. Cells can only be made visible by microscopes. Microrganisms are organisms that usually consist of one single cell. Because of this, they are often referred to as "single-celled organisms". They are so small, that humans cannot visualise them. We can only see them through microscopes, by which the cells are greatly enlarged.

At first, microorganisms were not seen as a separate kind. Microrganisms that carried out photosynthesis (see carbon cycle) were classified in the plant kingdom, and microrganisms that ingested food were placed in the animal kingdom. However, in the 19th century, scientists had identified a wide variety of microrganisms with diverse cell structures, very specific internal structures, and specific reproductive patterns that made them realize these organisms did not belong to the plant or animal kingdom.

What is the immunology?

Immunology is about understanding the development and function of our Immune System. Immunologists understand how our body fights infections and how we can use vaccination to protect ourselves against infectious diseases and cancer. Immunologists use all modern technologies in biology to unravel the inner details of the immune system, including genetics, molecular biology, biochemistry, cell biology, bioinformatics and immunogenetics and animal models.

While the immune system is made to protect us from infections, failures of the immune system can lead to a number of diseases with serious impact on quality of life, including immune deficiencies, autoimmune diabetes, arthritis, inflammatory bowl diseases, skin blistering diseases or cancer of cells in the immune systems, called lymphomas.

Full Microbiology Review

Table of Contents

1. Bacteria Review
Review of bacteria based on conceptual questions and answers. Study all questions to prepare yourself for exams.
2. Protists (Protozoans and Algae) Review
Questions and answers on protozoans and algae. Complete protists review for you to study and learn.
3. Fungi Kingdom Review
Study a full fungi review with our intelligent sequence of questions and answers.
4. Viruses Review
Study viral structure, reproduction, DNA viruses, RNA viruses and more. Read all questions and answers on viruses.
What kinds of microrganisms are there?
1. Basic Bacteriology
i. Bacterial Structure and Function
ii. Bacterial Growth, Nutrition, and Metabolism
iii. Normal Flora, Pathogenic Mechanisms, and Host Defense
iv. Bacterial Genetics
v. Mechanisms of Bacterial Control
vi. Mechanisms of Resistance
Review Questions
2. Clinical Bacteriology
i. Gram Positive Rods
ii. Gram Positive Cocci
iii. Gram Negative Cocci
iv. Gram Negative Rods
v. Anaerobic Bacterial
vi. Spirochetes and Chlamydia
vii. Rickettsial and Infections
viii. Mycobacteria and Mycoplasms
Review Questions
3. Basic Virology
i. Virus Structure and Classification
ii. Virus Replication
iii. Viral Genetics
iv. Viral Pathogenesis
v. Diagnostic Virology and Antiviral Therapy
Review Questions
4. Clinical Virology
i. DNA Viruses
ii. Negative Strand RNA Viruses
iii. Positive Strand RNA Viruses
iv. Double Strand RNA Viruses
v. Slow Viruses
vi. Retroviruses
Review Questions
5. Basic Mycology
i. Structure, Growth, and Laboratory Diagnosis
ii. Fungal Pathogenesis and Host Defense
iii. Prevention, Treatment, and Control
Review Questions
6. Clinical Mycology
i. Cutaneous and Subcutaneous Mycoses
ii. Systemic Mycoses
iii. Opportunistic Mycoses
Review Questions
7. Basic Parasitology
i. Structure, Growth, and Laboratory Diagnosis
ii. Parasitic Pathogenesis and Host Defense
iii. Prevention, Treatment, and Control
Review Questions
8. Clinical Parasitology
i. Protozoan Infections
ii. Worm Infections
Review Questions
9. Infectious Diseases
i. Central Nervous System Infections
ii. Gastrointestinal Tract Infections
iii. Respiratory Tract Infections
iv. Sexually Transmitted Diseases
v. Obstetric, Fetal, and Newborn Infections
vii. Vascular Infections
viii. Skin, Bone, and Joint Infections
ix. Eye, Ear, and Sinus Infections
x. Multisystem Infections
xi. Infections and Cancer
Review Questions
1 The science of microbiology
2 Cell structure
3 Classification of bacteria
4 The growth, survival, & death of microorganisms
5 Cultivation of microorganisms
6 Microbial metabolism
7 Microbial genetics
8 Immunology
9 Pathogenesis of bacterial infection
10 Antimicrobial chemotherapy
11 Normal microbial flora of the human body
12 Spore-forming gram-positive bacilli : bacillus & clostridium species
13 Non-spore-forming gram-positive bacilli : corynebacterium, propionibacterium, listeria, erysipelothrix, actinomycetes, & related pathogens
14 The staphylococci
15 The streptococci
16 Enteric gram-negative rods (enterobacteriaceae)
17 Pseudomonads, acinetobacters, & uncommon gram-negative bacteria
18 Vibrios, campylobacters, helicobacter, & associated bacteria
19 Haemophilus, bordetella, brucella, & francisella
20 Yersinia & pasteurella
21 The neisseriae
22 Infections caused by anaerobic bacteria
23 Legionellae, bartonella, & unusual bacterial pathogens
24 Mycobacteria
25 Spirochetes & other spiral microorganisms
26 Mycoplasmas & cell wall-defective bacteria
27 Rickettsia & ehrlichia
28 Chlamydiae
29 General properties of viruses
30 Pathogenesis & control of viral diseases
31 Parvoviruses
32 Adenoviruses
33 Herpesviruses
34 Poxviruses
35 Hepatitis viruses
36 Picornaviruses (enterovirus & rhinovirus groups)
37 Reoviruses, rotaviruses, & caliciviruses
38 Arthropod-borne & rodent-borne viral diseases
39 Orthomyxoviruses (influenza viruses)
40 Paramyxoviruses & rubella virus
41 Coronaviruses
42 Rabies, slow virus infections, & prion diseases
43 Human cancer viruses
44 AIDS & lentiviruses
45 Medical mycology
46 Medical parasitology
47 Principles of diagnostic medical microbiology
48 Cases & clinical correlations
BACTERIOLOGY
IMMUNOLOGY
VIROLOGY
PARASITOLOGY
MYCOLOGY

What are the five major structures found in ALL bacteria?
Draw a simple diagram of a bacterial cell, labeling all five structures.
What are the other five bacterial cell features that are only found in certain bacteria? Name each of them and describe its function.
What are the differences between a bacterial chromosome and a plasmid?
Bacteria reproduce through a process called binary fission. Is this a sexual or asexual process?
Conjugation is a form of genetic recombination in bacteria. We referred to it as “bacteria sex,” but this is not really an appropriate name for the process. Why not?
Describe the three ways in which bacteria undergo genetic recombination.
There are several ways in which bacteria can obtain nutrients. Describe at least three of them, using the proper terms.
Who is the current quarterback for the Detroit Lions? (Just kiddingÂ… you donÂ’t really have to answer that)
What is the difference between an aerobe and an anaerobe? What do you think is meant by the term facultative (as in “facultative anaerobe”)?
Name the three special types of archaebacteria we talked about in class. What is so special about each one?
What are six major categories we use to categorize (classify) eubacteria? Can you think of any others we have talked about?
What is the (structural) difference between a gram(+) and gram(-) bacterium? How does this help us to tell the difference between them in a gram stain?
Why would doctors need to perform gram stains on the bacteria they take from a sick personÂ’s body?
My doctor correctly prescribed amoxicillin (an antibiotic) for me when I was sick. Did I have a bacterial or viral infection? How can you tell?
How are antibiotics produced?
Meijer pharmacies recently unveiled a new plan to provide free antibiotics for children. Although this may be beneficial to families without medical insurance, it could lead to some other problems, such as the overuse of antibiotics. What are some potential dangers to this plan?
Bacterial infections are often caused by toxins released by the bacteria. List the general characteristics of the two types of toxins discussed in class. (What type of bacterium releases it? When is it released? What is it made of?)
Distinguish between the three major shapes of bacteria. Draw a picture of each and label it.
Name at least three differences between bacteria and viruses.

Questions 1. What do bacteria look like?
2. What are the three basic shapes?
3. How are bacteria classified? Name the two large classes.
4. Where have bacteria been located? Name as many places as possible.
5. How many bacteria live on Earth? (The third Web site listed above has information on this.)
6. How quickly do bacteria reproduce? Try to show this pattern as a graph or other visual display.
7. What do bacteria eat? How do they take in their food?

Questions

1. Name some helpful bacteria.
2. How are bacteria helpful to humans? Give at least four examples.
3. How do bacteria help keep ecosystems healthy?
4. What do bacteria do that is especially helpful to plants?

Questions

1. What are pathogenic bacteria?
2. Where do these harmful bacteria usually live?
3. Describe how bacteria can cause food poisoning.
4. Give two examples of bacteria that cause disease. Include their names and how the bacteria spread.
5. How can these diseases be prevented?

Discussion Questions

1. What was the single most surprising fact you learned about bacteria? For example, did you know that bacteria live in your body? How have your ideas about bacteria changed as a result of working on this lesson?
2. You may have read in the newspaper or heard your parents talking about bacteria that are resistant to antibiotics. This means that streptococcus or harmful bacteria may not die when a person takes an antibiotic. Based on what you have learned about bacteria, why do you think they can become resistant?
Hint: Remember how quickly bacteria reproduce. (BacteriaÂ’s rapid reproduction increases the likelihood that a particularly hardy type of bacteria will emerge. This bacteria will then reproduce, resulting in greater numbers of hardy bacteria. This situation is compounded by overuse of antibiotics, which enables bacteria to develop mutations against certain medications.)
3. What do you think microbiologists, or scientists who study bacteria, look for in their research? (Answers could include the discovery of different kinds of bacteria, the conditions in which bacteria grow, and the rate at which bacteria multiply.) Why is it important to continue to learn about bacteria? (New strains of bacteria are always emerging, and bacteria can become resistant to antibiotics. It is important for scientists to be aware of these and other developments.)

- What is an antibody?
- What are antibodies used for?
- How are they produced?
- How to determine neutralization amounts for peptide/antibodies?
- Will an antibody cross-react with a different species not listed in the catalog?
- What is the shelf life?
- What is the dynamic range/standard curve?
- What is the assay time?
- What type of sample can be measured but does not require extraction?
- Can components from different lots be combined?
- Can the diluted antisera for RIA or the antibody in RIA or EIA kits be used for staining?
- What is the dilution for immunohistochemistry antisera?
How are allergies diagnosed?
How are allergy tests done?

What is an antibody?

Antibodies are host proteins produced in response to the presence of foreign bodies, infectious organisms and their toxic products. They are synthesized primarily by plasma cells and circulate throughout the blood and lymph where they bind specifically to foreign molecules (antigens) and speed its removal from the host.

What are antibodies used for?

The interaction of an antibody with an antigen forms the basis of all immunochemical techniques. Antibodies are produced to identify specific antigens. Their affinity to specific antigens aid in learning more about peptides and proteins. Some applications include:

- Quantification: Radioimmunoassay and enzyme immunoassay.
- Localization: Immunostaining via avidin/streptavidin.
- Neutralization of peptides and other antigens.
- Precipitation of peptides and other antigens.

How are they produced?

Antibodies are produced by plasma cells. The host's response to an antigenic presence can be characterized as two responses: the primary response and the secondary response. The primary response is a relatively weak response to the initial antigenic exposure. A series of specialized events during the primary response prepare the host for subsequent exposure to the same antigen. When the antigen is reintroduced, the secondary response is relatively more rapid and intense. To fully optimize antibody production, the peptide/antigen must be prepared in a way to induce the strongest and most appropriate response. Since most peptides are too small to induce an adequate response, the peptides are normally conjugated to protein carriers (such as KLH, thyroglobulins, BSA, etc.). This larger complex boosts the response. Antibodies harvesting usually takes place within six months.

How to determine neutralization amounts for peptide/antibodies?

To determine the amount of peptide needed to neutralize 1 ml of neat, undiluted antiserum: 1ml of antiserum contains 10mg of IgG. Among the 10mg total IgG, only 1% (under estimate) is against the specific antigen, therefore: (10mg)(1%)= 0.1mg or 100 mg of specific IgG against the antigen.

Molecular weight of IgG=150,000 daltons, therefore, 100mg of IgG/150,000= 0.00066mmol or 0.66nmol 0.66nmol of antibody can neutralize twice the amount of peptide or 1.32nmol of specific peptide. Weight of peptide needed to neutralize = (1.32nmol)(M.W. of peptid)=X ng of peptide needed.

To determine the amount of antibody needed to neutralize 1mg of peptide: 1mg of peptide/MW of peptide=X mmol (X times 1000nmol)1000 nmol of peptide needs 1000/2 nmol of specific IgG to be neutralized 1000/2 nmol of IgG= (1000/2 nmol)(MW of IgG)=Y ng of specific IgG. Since Y ng is the minimum needed, we recommend an overdose amount to be determined by end user.

Will an antibody cross-react with a different species not listed in the catalog?

An antibody may cross react with peptide of a different species. Our antibodies are raised against a specific synthetic peptide sequence and if the sequence of another species is identical or highly similar to the sequence of our synthetic peptide, the possibility of recognition is high. For peptide sequences, please check our website, Bachem catalog, or call your nearest Bachem sales office. In the case of unestablished cross reactivity of a sequence from an untested species, we cannot guarantee recognition by the antibody. However, we do offer cross reactivity testing upon request.

What is the shelf life?

All the RIA kits have an expiration date of 6 weeks after preparation which is the expiration date of the tracer (125I-peptide). The other components such as RIA buffer, goat anti-rabbit IgG (GARGG), normal rabbit serum (NRS), antibody, and standard are stable for 6 months from the day of receipt when stored at 2-8°C. Once rehydrated, there is no guarantee on any of these components. Two weeks is the expected cutoff point for viability after rehydration.

All the EIA kits have an expiration date of 4 months from shipping date. Lyophilized antisera are guaranteed for 1 year from shipping date at -20°C.

What is the dynamic range/standard curve?

For RIA kits, we have three ranges: 0.1-64pg/tube, 1-128 pg/tube, and 10-1280 pg/tube. For EIA kits, we have two ranges: 0-10 ng/ml and 0-25 ng/ml. For your specific kit, please check your datasheet.

What is the assay time?

For RIA and RIA kits, extraction-free - 3 days (2 overnight incubation) (not including extraction procedure). For EIA kit - 4.5 hours (not including extraction procedure). For EIA kits, extraction-free - overnight due to incubation time. For staining kits - 3.5 hours to 2 days.

What type of sample can be measured but does not require extraction?

Sample extraction is strongly recommended for all RIA and EIA kits. For the extraction-free kits, if the peptide level in the sample to be measured is lower than the sensitivity of the kit, the extraction procedure may still be necessary to concentrate the sample.

Can components from different lots be combined?

Each lot is specifically formulated to give optimal sensitivity. Therefore, we do not recommend combining components with different lot numbers. In addition, we cannot guarantee the performance of our products when used outside of our protocol.

Can the diluted antisera for RIA or the antibody in RIA or EIA kits be used for staining?

The diluted antisera for RIA and the antibody for RIA or EIA kits are too diluted for staining purpose. They should not be used for staining.

What is the dilution for immunohistochemistry antisera?

For immunofluorescence staining, the suggested starting dilution is 1:200. For PAP or ABC method, it is 1:500 to 1:1000.

An antigen is floating around the extracellular fluid of the body. Antibodies specific for that antigen attach to the antigen. Name the two antibody interactions that could now occur. Explain the different cells involved in these two interactions and how it leads to the exposal of the antigen.

5) Define Antigen, Pathogen, Antibody, Immunoglobulin, vaccination, and immunization.

6) What is the difference between innate immunity and adaptive immunity? What is the difference between humoral and cellular?

7) Draw and label a diagram of a tube of blood that has anticoagulants. Draw and compare this to a tube of centrifuged blood that has no anticoagulant.

8) What is the blood composed of? What are the percentages in normal human blood?

9) List the tissues of the lymphoid system. Which ones are considered primary tissues? Which are considered secondary?

10) Make a table listing all of the immune cells discussed in lecture. Use the format below: Immune cell

Histology

Precursor(s)

Locations

Classifications

Innate or adaptive

Basic functions/targets

T cell

Large, round nucleus

Lymphoid precursor

Pluripotent stem cell

Mature in Thymus

Circulate in blood and home to sites of infection

Lymphocyte

Mononuclear

Mostly adaptive

Cytotoxic killing

Helper cells

1. Describe the differences between polymorphonuclear cells and lymphocytes.
2. What are antigen presenting cells and name the three antigen presenting cells?
3. What are the main differences between the innate and adaptive immune response?
4. What is the difference between the humoral and cell-mediated immune response and which cells are involved in these different responses?
5. What are the main white blood cells involved in the a) innate immune response and b) adaptive immune response?
6. What are the main differences between T and B-cell antigen receptors?
7. What is clonal selection?
8. Describe the two mains the innate immune system uses to kill pathogens?
9. Describe the process of antibody production from activation of the B-cell to the production of immunoglobulin?
10. Describe the actions of the different T-lymphocytes.

A patient with rheumatoid arthritis presents to her physician and mentions that after many years without teeth problems, she has recently developed seven caries. This is a clue to her clinician that she should be evaluated for which of the following diseases?

A. Oral squamous cell carcinoma

B. Polyarteritis nodosa

C. Sjögren's syndrome

D. Systemic lupus erythematosus

E. Thyrotoxicosis

Explanation:

The correct answer is C. Rheumatoid arthritis can coexist with a variety of autoimmune diseases (including those listed in the answers), but is most frequently associated with Sjögren's syndrome. Sjögren's syndrome is due to autoimmune involvement with subsequent scarring of the salivary and lacrimal glands, leading to dry eyes and dry mouth. Secondary effects include parotid gland enlargement, dental caries, and recurrent tracheobronchitis.

Squamous cell carcinoma of the mouth (choice A) is not associated with dryness of the mouth.

Polyarteritis nodosa (choice B) is a systemic necrotizing vasculitis. Patients present with low-grade fever, weakness, and weight loss. They may also have abdominal pain, hematuria, renal failure, hypertension, and leukocytosis.

Systemic lupus erythematosus (choice D) is an autoimmune disease characterized by vasculitis (which may produce a variety of symptoms depending on the site of the lesion), rash, renal disease, hemolytic anemia, and neurologic disturbances.

Thyrotoxicosis (choice E) produces insomnia, weight loss, tremors, heat intolerance, excessive sweating, and frequent bowel movements or diarrhea.

A 32-year-old, blood type A positive male receives a kidney transplant from a blood type B positive female donor with whom he had a 6-antigen HLA match. Once the kidney is anastomosed to the man's vasculature, the transplant team immediately begins to observe swelling and interstitial hemorrhage. After the surgery, the patient developed fever and leukocytosis and produced no urine. Which of the following is the most likely explanation?

A. Acute rejection due to antibody-mediated immunity

B. Acute rejection due to cell-mediated immunity

C. Chronic rejection due to cell-mediated immunity to minor HLA antigens

D. Hyperacute rejection due to lymphocyte and macrophage infiltration

E. Hyperacute rejection due to preformed ABO blood group antibodies

Explanation:

The correct answer is E. The patient is suffering from hyperacute rejection due to the preformed anti-B ABO blood group antibody found in all type A positive individuals. Hyperacute rejection occurs within minutes to a few hours of the time of transplantation, and is due to the destruction of the transplanted tissue by preformed antibodies reacting with antigens found on the transplanted tissue that activate complement and destroy the target tissue. Preformed antibodies can also be due to presensitization to a previous graft, blood transfusion, or pregnancy.

Acute rejection due to antibody-mediated immunity (choice A) is incorrect because this patient suffered from hyperacute rejection (immediate) occurring within minutes to hours, rather than days.

Acute rejection due to cell-mediated immunity (choice B) will not occur until several days or a week following transplantation. Acute rejection is due to type II and type IV reactions.

Chronic rejection, due to the presence of cell-mediated immunity to minor HLA antigens (choice C), occurs in allograft transplantation months to even years after the transplant. Chronic rejection is generally caused by both humoral and cell-mediated immunity.

An accelerated acute rejection, occurring in 3-5 days, can be caused by tissue infiltration and destruction by presensitized T lymphocytes and macrophages (choice D) and/or antibody-dependent, cell-mediated cytotoxicity (ADCC). Note that this is not a hyperacute reaction.

A 42-year-old auto mechanic has been diagnosed with end-stage renal disease. His identical twin brother has the same HLA alleles at all loci, and volunteers to donate a kidney to his brother. Which of the following terms correctly describes the proposed organ transplant?

A. Allograft

B. Autograft

C. Heterograft

D. Syngeneic graft

E. Xenograft

Explanation:

The correct answer is D. A syngeneic graft is the transfer of tissue between genetically identical individuals (identical twins). This type of graft is usually successful. An allograft (choice A), or homograft, is a graft between genetically different members of the same species.

This type of graft would be between two different humans and would most likely be rejected unless the recipient is given immunosuppressive drugs.

An autograft (choice B) is a transfer of an individual's own tissue to his or her own body and is nearly always successful.

A heterograft (choice C) is the old term for a xenograft. This transfer of tissues between different species is not generally successful.

A xenograft (choice E) is a transfer of tissue between different species and is always rejected except for a few exceptions (e.g., pig heart valve).

Loss of which of the following classes of molecules on the surface of a tumor cell target would result in loss of susceptibility to killing by host immune cells?

A. CD3

B. CD4

C. CD8

D. MHC class I

E. MHC class II

Explanation:

The correct answer is D. After the MHC class I molecule has moved to the surface of the tumor cell, peptide fragments from the tumor are presented in a groove of the class I molecule. The peptide fragments are presented to cytotoxic CD8 T cells, which recognize the MHC class I molecules on the cell surface and kill the tumor cell. Loss of this molecule would therefore prevent the tumor cell from being killed.

The CD3 molecule (choice A) is a marker on all T cells. It is involved in signal transduction, but not antigen recognition. This molecule would not be on the surface of tumor cells.

The CD4 molecule (choice B) is not on the surface of a tumor cell, but it is on the surface of a CD4+ T helper lymphocyte.

The CD8 molecule (choice C) is not on the surface of a tumor cell, but it is on the surface of a CD8+ cytotoxic T lymphocyte.

MHC class II antigens (choice E) are not involved in killing of tumor cell targets. They present peptide fragments (derived from intracellular killing of extracellular organisms by macrophages) to CD4 T lymphocytes.

A 44-year-old white female presents with severe Raynaud's phenomenon, dysphagia, sclerodactyly, and facial and palmar telangiectasias. Which of the following autoantibodies is most likely to be present in this patient?

A. Anti-centromere antibody

B. Anti-histone antibody

C. Anti-SS-A

D. Anti-SS-B

E. Rheumatoid factor

Explanation:

The correct answer is A. The symptoms described in the question are classic for CREST syndrome. CREST syndrome is the combination of calcinosis, Raynaud's phenomenon, esophageal dysmotility, sclerodactyly, and telangiectasia. Anti-centromere antibodies are reported in a high percentage of patients with CREST, but in a small percentage of pure scleroderma patients.

Anti-histone antibody (choice B) is a marker for drug-induced lupus erythematosus. Anti-SS-A (choice C) is a marker for Sjögren's syndrome, characterized by dry eyes and dry mouth.

Anti-SS-B (choice D) is an autoantibody directed against ribonucleoproteins, and is a marker for Sjögren's syndrome.

Rheumatoid factor (choice E) is generally an IgM autoantibody directed against the Fc portion of IgG. RF is positive in 80% of patients with rheumatoid arthritis and may also be found in low titers in patients with chronic infections, other autoimmune diseases such as SLE and Sjögren's syndrome, or chronic pulmonary, hepatic, or renal diseases.

Which of the following is a major interleukin produced by CD4+ T helper 1 (TH1) lymphocytes?

A. IL-1

B. IL-2

C. IL-4

D. IL-6

E. IL-8

Explanation:

The correct answer is B. IL-2 produced by TH1 cells stimulates natural killer (NK) cells and CD8+ T cytotoxic lymphocytes and combines with IL-2 receptors on the TH1 cells to cause "self-stimulation". IL-2 is also involved in downregulating CD4+ T helper 2 lymphocytes.

IL-1 (choice A) is derived from macrophages and is a major inflammatory molecule.

IL-4 (choice C) is derived from CD4 + T helper 2 cells. It acts to downregulate CD4 + TH1 cells and allows class switching to IgE.

IL-6 (choice D) is derived from macrophages. This molecule stimulates acute phase protein production by the liver, as well as the production of other opsonizing molecules.

IL-8 (choice E) is derived from macrophages and is a major chemotactic molecule for neutrophils.

Maximal ventricular Na+ channel conductance occurs during which phase of the ECG?

A. P wave

B. QRS interval

C. ST interval

D. T wave

E. U wave

Explanation:

The correct answer is B. Phase 0 of the cardiac muscle action potential (AP) corresponds to the opening of voltage-dependent sodium channels, causing a transient but large increase in sodium conductance during ventricular depolarization. The shape of the QRS complex of the ECG is determined by the spread of the combined phase 0 (depolarization) of all the ventricular muscle of the heart.

The P wave (choice A) corresponds to atrial depolarization.

The ST interval (choice C) represents the time interval during which all ventricular cells are in phase 2 of their AP. Phase 2 is dominated by a high, prolonged calcium conductance through slow channels. The length of the ST interval corresponds closely to the AP duration in ventricular muscle.

The T wave (choice D) corresponds to ventricular repolarization.

The U wave (choice E) is found only occasionally in ECGs and is presumed to be due to the repolarization of papillary muscle.

Antigens processed by the exogenous antigen presentation pathway are presented in association with which of following?

A. Fc receptors

B. IgG heavy chains

C. MHC class I molecules

D. MHC class II molecules

E. T cell receptor (TCR)

Explanation:

The correct answer is D. When pathogenic organisms are phagocytized and degraded in the exogenous antigen presentation pathway, the antigenic molecules are presented on the surface of the antigen-presenting cell by MHC class II molecules to a CD4+ T lymphocyte with a specific TCR for the specific antigenic epitope.

The Fc (choice A) portion of an antibody molecule is the part of the immunoglobulin that attaches to the Fc receptors on phagocytic cell surfaces. When a Fab portion of the antibody is attached to the pathogen and the Fc attaches to the phagocytic cell surface, the phagocyte can destroy the pathogen more efficiently.

The IgG molecule (choice B) is an immunoglobulin that reacts with the antigen after it has been destroyed and presented to the T cell. The IgG immunoglobulin is never involved in antigen presentation.

In the endogenous antigen presentation pathway (eg, a virus infecting a cell), the cell would display epitopes from the virus in association with class I molecules (choice C) to the CD8+ cytotoxic T cell.

The TCR (choice E) is the area of the mature T cell that reacts with the antigen epitope that is presented by the antigen-presenting cell.

Loss of which of the following classes of molecules on the surface of a tumor cell target would result in loss of susceptibility to killing by host immune cells?

A. CD3

B. CD4

C. CD8

D. MHC class I

E. MHC class II

Explanation:

The correct answer is D. After the MHC class I molecule has moved to the surface of the tumor cell, peptide fragments from the tumor are presented in a groove of the class I molecule. The peptide fragments are presented to cytotoxic CD8 T cells, which recognize the MHC class I molecules on the cell surface and kill the tumor cell. Loss of this molecule would therefore prevent the tumor cell from being killed.

The CD3 molecule (choice A) is a marker on all T cells. It is involved in signal transduction, but not antigen recognition. This molecule would not be on the surface of tumor cells.

The CD4 molecule (choice B) is not on the surface of a tumor cell, but it is on the surface of a CD4+ T helper lymphocyte.

The CD8 molecule (choice C) is not on the surface of a tumor cell, but it is on the surface of a CD8+ cytotoxic T lymphocyte.

MHC class II antigens (choice E) are not involved in killing of tumor cell targets. They present peptide fragments (derived from intracellular killing of extracellular organisms by macrophages) to CD4 T lymphocytes.

A college student sitting in the stands at a football game suddenly begins breathing hard and complains to his friends of tightness in his chest. Minutes later, he is sweating profusely and faints. It is discovered that he had been stung by a bee. Paramedics arrive, assess the situation, then successfully treat the young man. Which one of the following drugs was most likely initially administered in this case?

A. Diphenhydramine

B. Blocking antibody

C. Cromolyn sodium

D. Epinephrine

E. Theophylline

Explanation:

The correct answer is D. This college student is experiencing a major anaphylactic reaction (type I hypersensitivity reaction) with associated bronchoconstriction and shock. Epinephrine is the treatment of choice for anaphylaxis. It will relax the smooth muscle of the respiratory tract and stimulate the heart to restore cardiac output. Epinephrine also prevents mast cell degranulation by increasing cyclic AMP levels.

Diphenhydramine (choice A), an H1 histamine receptor antagonist, is a good drug for mild allergic rhinitis, but would be ineffective in anaphylaxis.

Blocking antibody (choice B) is IgG antibody that has been produced by the patient in response to an allergen over a long period of stimulation (e.g., during desensitization by an allergist). This procedure is excellent for several different types of allergens, but not for a systemic material such as bee venom.

Cromolyn sodium (choice C) stabilizes mast cell membranes, thereby inhibiting degranulation and histamine release. This is an excellent drug if used to prevent an acute reaction to a known allergen, but it would not be used for an anaphylactic reaction.

Theophylline (choice E) inhibits phosphodiesterase, increasing cyclic AMP levels, and thereby inhibiting mast cell degranulation. This is an excellent drug for asthma and for long term allergy treatment especially in children, but would not be used for anaphylaxis.

A traveler to a foreign country develops acute lymphatic filariasis four months after his return to the United States. His symptoms include scrotal inflammation, itching, and localized scrotal swelling and tenderness of the inguinal lymph nodes. Which of the following immune mechanisms does the body employ against the live filarial worms ?

A. Antibody-dependent cell-mediated cytotoxicity

B. Anti-receptor antibodies

C. Arthus reaction

D. Complement-mediated reactions

E. Deposition of circulating antigen-antibody complexes

Explanation:

The correct answer is A. Filarial parasites have a complex life cycle beginning with transmission by mosquitoes or other arthropods. The offspring of adults (microfilariae) either circulate in the blood or migrate through the skin, often inhabiting lymphatics. Microfilariae are ingested by the arthropod vector and develop over 1 to 2 weeks into new infective larvae. The adult worms elicit an inflammatory reaction in the lymphatics, eventually leading to lymphatic obstruction and edema. The body reacts to large tissue parasites, such as filarial worms, by coating them with a thin layer of IgE molecules, which trigger eosinophil-mediated cytotoxicity (a form of Type II hypersensitivity) and release of vasoactive and spasmogenic substances from mast cells and basophils (local anaphylaxis, a form of Type I hypersensitivity).

Other forms of Type II hypersensitivity include complement-mediated reactions (choice D, e.g., the lysis of blood cells seen in transfusion reactions) and anti-receptor antibodies (choice B, e.g., muscle weakness in myasthenia gravis).

Type III hypersensitivities usually take the form of vasculitis secondary to deposition of circulating antigen-antibody complexes (choice E); the Arthus reaction (choice C) is a specific variant of these reactions in which local vasculitis induces tissue necrosis, often in the skin.

A 7-month-old child is hospitalized for a yeast infection that does not respond to therapy. The patient has a history of multiple, acute pyogenic infections. Physical examination reveals that the spleen and lymph nodes are not palpable. A differential WBC count shows 95% neutrophils, 1% lymphocytes, and 4% monocytes. A bone marrow biopsy contains no plasma cells or lymphocytes. A chest x-ray reveals the absence of a thymic shadow. Tonsils are absent. These findings are most consistent with

A. Bloom's syndrome

B. chronic granulomatous disease

C. severe combined immunodeficiency

D. Waldenström's macroglobulinemia

E. Wiskott-Aldrich syndrome

Explanation:

The correct answer is C. Severe combined immunodeficiency (SCID) is associated with deficiencies in both B and T cells due to a defect in differentiation of an early stem cell. Over 50% of the cases are caused by a gene defect on the X chromosome, resulting in a defective IL-2 receptor. The disease may exhibit a sex-linked or an autosomal recessive pattern of inheritance. The autosomal recessive variant is characterized by a deficiency of adenosine deaminase, which results in accumulation of metabolites that are toxic to both B and T stem cells in the bone marrow. Children usually die within the first 2 years of life with severe infections unless they receive bone marrow transplants.

Bloom's syndrome (choice A) is an autosomal recessive disorder included in the chromosomal instability group of syndromes. It is associated with small body size, immunodeficiency, light-sensitive facial erythema, and a major predisposition to cancer.

Chronic granulomatous disease (choice B) is caused by a deficiency of NADPH oxidase in neutrophils, resulting in loss of the first step of the myeloperoxidase system, and an absence of the respiratory burst. Patients are susceptible to staphylococcal infections and granulomatous infections.

Waldenström's macroglobulinemia (choice D) is a monoclonal gammopathy characterized by high serum levels of IgM and hyperviscosity complications.

Wiskott-Aldrich syndrome (choice E) is an immunodeficiency syndrome characterized by thrombocytopenia, eczema, and recurrent sinopulmonary infections. The patient has low levels of IgM and increased levels of IgG, IgA, and IgE.

A 27-year-old white male presents with a 3-week history of several swollen and painful toes and knees. He has a past history of conjunctivitis. He also describes some low back stiffness that is more severe in the morning. Which of the following is the most likely diagnosis?

A. Gout

B. Lyme disease

C. Reiter's syndrome

D. Rheumatoid arthritis

E. Septic arthritis

Explanation:

The correct answer is C. This is a case of Reiter's syndrome. Patients typically present with the acute onset of arthritis (usually asymmetric and additive), with involvement of new joints occurring over a period of a few days to 2 weeks. Joints of the lower extremities are the most commonly involved, but wrists and fingers can also be affected. Dactylitis (sausage digit), a diffuse swelling of a solitary finger or toe, is a distinctive feature of Reiter's arthritis and psoriatic arthritis. Tendonitis and fasciitis are common. Spinal pain and low back pain are common. Conjunctivitis, urethritis, diarrhea, and skin lesions are also associated with Reiter's syndrome. Up to 75% of patients are HLA-B27 positive. Microorganisms which can trigger Reiter's syndrome include Shigella spp., Salmonella spp., Yersinia spp., Campylobacter jejuni, and Chlamydia trachomatis. Most patients are younger males.

Gout (choice A) usually presents as an explosive attack of acute, very painful, monarticular inflammatory arthritis. Hyperuricemia is the cardinal feature and prerequisite for gout. The first metatarsophalangeal joint is involved in over 50% of first attacks. Lyme disease (choice B), caused by Borrelia burgdorferi, presents with a red macule or papule at the site of the tick bite. This lesion, called erythema chronicum migrans, slowly expands to form a large annular lesion with a red border and central clearing. The lesion is warm, but usually not painful. The patient also has severe headache, stiff neck, chills, arthralgias, and profound malaise and fatigue. Untreated infection is associated with development of arthritis. The large joints (e.g., knees) are usually involved with the arthritis lasting for weeks to months.

Rheumatoid arthritis (choice D) begins insidiously with fatigue, anorexia, generalized weakness, and vague musculoskeletal symptoms leading up to the appearance of synovitis. Pain in the affected joints, aggravated by movement, is the most common manifestation of established rheumatoid arthritis. Generalized stiffness is frequent and is usually greatest after periods of inactivity. Morning stiffness of greater than 1 hour in duration is very characteristic. Rheumatoid arthritis is more common in females. The metacarpophalangeal and proximal interphalangeal joints of the hands are characteristically involved.

Septic arthritis (choice E) is caused by a variety of microorganisms, including Neisseria gonorrhoeae and Staphylococcus aureus. Hematogenous spread is the most common route in all age groups. 90% of patients present with involvement of a single joint, usually the knee. The usual presentation is moderate-to-severe pain, effusion, muscle spasm, and decreased range of motion. Peripheral leukocytosis and a left shift are common. Disseminated gonococcal infections present as fever, chills, rash, and articular symptoms. Papules progressing to hemorrhagic pustules develop on the trunk and extensor surfaces of the distal extremities. Migratory arthritis and tenosynovitis of multiple joints is common.

A researcher develops a specific antibody to the complement component C3b. Assume that intravenous administration of the antibody prevents the biological effects of C3b. Administration of the antibody would be expected to interfere with which of the following biological functions?

A. Decreased appetite

B. Fever

C. Increased collagen synthesis by fibroblasts

D. Increased leukocyte adherence to endothelium

E. Opsonization to facilitate phagocytosis

Explanation:

The correct answer is E. C3b acts as an opsonin, along with IgG, to facilitate phagocytosis. None of the other functions listed in the answer choices is attributable to C3b. Instead, all of the other answer choices are functions of the cytokines interleukin-1 (IL-1) and tumor necrosis factor (TNF). Tumor necrosis factor is produced by macrophages and activated T-cells, while many different cell types (including macrophages) produce IL-1.

Decreased appetite (choice A) is typical during an acute illness (such responses are called acute phase reactions). Other acute phase reactions include fever (choice B), increased sleep, shock, leukocytosis, and increased serum acute phase proteins.

Fibroblasts proliferate and increase their synthesis of collagen (choice C), collagenase, protease, and prostaglandin E in response to IL-1 and TNF.

Effects of IL-1 and TNF on endothelium include increased synthesis of mediators (prostaglandins, IL-1, IL- 8, platelet-derived growth factor), increased leukocyte adherence (choice D), and increased procoagulant activity with decreased anticoagulant activity.

A 22-year-old female comes to the sexually transmitted disease (STD) clinic for her first visit. She tells the nurse practitioner that she has had four different sexual partners in the last six months and only one of them used a condom. She also admits that she used IV drugs on several occasions two years ago. She notes fever, weight loss, lack of appetite, and periodic difficulty breathing over the past few months. She has an HIV test performed, which is positive. The physician decides to do a confirmatory test for HIV. Which one of the following tests would the physician order?

A. ELISA (Enzyme-linked immunosorbent assay)

B. FACS ( Fluorescence activated cell sorting)

C. RAST (Radioallergosorbent Test)

D. RID (Radial immunodiffusion)

E. Western blot

Explanation:

The correct answer is E. The Western blot is the most appropriate test for confirmation of HIV infection. It identifies several different antibodies against HIV (anti- gp120, -gp41, -p24, and -p17).

The initial HIV test this patient had was most likely an ELISA. The ELISA (choice A) can be used to detect p24 antibody in the patient, but is not as specific as the Western blot.

Fluorescence activated cell sorting (choice B) is a technique used to separate and count specific numbers and types of cells in a sample. An example of this would be to count the number of B cells and T cells in a specific blood sample.

RAST testing (choice C) is used to determine the level of specific IgE present in a patient that reacts with a specific allergen that has been applied to a disk or glass bead.

Radial immunodiffusion (choice D) is an excellent test used for quantitation of immunoglobulin levels in patients.

This is used to determine the IgG, IgM and IgA levels in patient's serum. This test cannot be used to measure levels of IgD or IgE because these two immunoglobulins are at such low levels a more sensitive test such as RIA (radioimmunoassay) or EIA (enzyme-linked immunoassay) must be used.

A 38-year-old woman complains of cold, painful fingertips, as well as difficulty swallowing and indigestion. Physical examination is remarkable for a thickened, shiny epidermis over the entire body, with restriction of movement of the extremities, particularly the fingers, which appear claw-like. Which of the following autoantibodies is likely to be found in this patient's serum?

A. Anti-DNA topoisomerase I (anti-Scl-70)

B. Anti-double-stranded DNA (ds DNA)

C. Anti-IgG

D. Anti-Sm

E. Anti-SS-A

Explanation:

The correct answer is A. This patient is suffering from systemic sclerosis, also called scleroderma. Antibodies to topoisomerase I (anti-Scl-70) occur in up to 70% of patients with diffuse systemic sclerosis, but only rarely in other disorders. Systemic sclerosis is characterized initially by excessive fibrosis and edema of the skin, especially the hands and fingers, producing sclerodactyly (characteristic changes in the fingers, which resemble claws). Raynaud's phenomenon is common. The diffuse type of systemic sclerosis generally spreads to include visceral organs such as the esophagus (producing dysphagia), the lungs (producing pulmonary fibrosis), the heart (leading to heart failure or arrhythmia), and the kidneys (renal failure causes 50% of scleroderma deaths). Females are affected more than males (3:1 ratio). A more restricted variant of systemic sclerosis with a somewhat more benign course is CREST syndrome (Calcinosis, Raynaud's syndrome, Esophageal dysmotility, Sclerodactyly, and Telangiectasia), characterized by the presence of anti-centromere antibodies (although 10% of CREST patients will have anti-topoisomerase antibody as well).

Anti-ds DNA (choice B) is characteristic of systemic lupus erythematosus, but is not common in patients with systemic sclerosis.

Rheumatoid factor is an autoantibody directed against IgG (choice C). It is found in patients with rheumatoid arthritis.

Anti-Sm (Smith antigen; choice D) is also characteristic of SLE rather than systemic sclerosis.

Anti-SS-A (choice E) is typically seen in Sjögren's syndrome (although it may also be seen in SLE).

A 4-year-old boy presents to the emergency room with muscle spasms. His past medical history is significant for recurrent infections and neonatal seizures. Evaluation of his serum electrolytes reveals hypocalcemia. This patient would be most susceptible to which of the following diseases?

A. Chickenpox

B. Diphtheria

C. Gas gangrene

D. Gonorrhea

E. Tetanus

Explanation:

The correct answer is A. This boy has DiGeorge's syndrome, as evidenced by his tetany (muscle spasms) due to hypocalcemia and his history of recurrent infections and neonatal seizures. The syndrome occurs because of an embryonic failure in the development of the third and fourth pharyngeal pouches. Patients have both hypoplastic parathyroids (producing hypocalcemia) and thymuses (producing T-cell deficiency and recurrent infections). Since cell-mediated immunity (which depends on T cells) is important in defense against infections caused by intracellular pathogens (such as viruses), patients with DiGeorge's are particularly susceptible to viral infections, such as chickenpox (varicella). They also have trouble with fungal pathogens (e.g., Candida) and mycobacteria.

Diphtheria (choice B) is caused by Corynebacterium diphtheriae, which produces disease by the elaboration of a very potent exotoxin. Therefore, humoral immunity (antitoxin), which is not usually compromised in DiGeorge's patients, is essential for defense against the organism. (Note that the C. diphtheriae exotoxin acts by causing the ADP-ribosylation of elongation factor-2 of eukaryotic cells, thereby inhibiting protein synthesis). The disease can be avoided by immunization with diphtheria toxoid.

Gas gangrene (choice C) is caused by Clostridium perfringens, which produces a potent alpha toxin that injures cell membranes. Therefore, humoral immunity would again play a predominant role in defense against this organism. Note that the disease occurs in wounds and would not be expected in an uninjured 4-year-old boy.

Gonorrhea (choice D) is caused by Neisseria gonorrhoeae and would not be expected in a 4-year-old boy unless there was evidence of sexual abuse. Virulence factors of this organism include pili, cell wall endotoxin and outer membrane, and secretory IgA protease. Antibody responses, neutrophils, and complement are of prime importance in defense against gonococcal infections.

Tetanus (choice E) is caused by Clostridium tetani and serves as a tricky distracter, as you might have quickly associated the patient's muscle spasms with this answer choice. (This is why it is important to read the question stem carefully before prematurely jumping to the responses). C. tetani, which gains entry through deep wounds, produces tetanus toxin (exotoxin) and can be prevented by immunization with tetanus toxoid.

A 30-year-old woman presents to a physician with a prominent rash over her nose and cheeks. She also has complaints of fever, malaise, and muscle soreness of several months duration. Serologic studies demonstrate positive ANA with autoantibodies to double-stranded DNA. This patient's probable condition is associated with which of the following HLA type(s)?

A. HLA-A3

B. HLA-B27

C. HLA-DR2 and HLA-DR3

D. HLA-DR3 and HLA-DR4

E. HLA-DR4

Explanation:

The correct answer is C. The disease is systemic lupus erythematosus, which is an autoimmune disorder associated with HLA-DR2 and HLA-DR3. The presentation described in the question stem is classic; patients without the characteristic malar or "butterfly" rash are much harder to diagnose because their complaints are initially typically very vague.

Associate HLA-A3 (choice A) with primary hemochromatosis.

Associate HLA-B27 (choice B) with psoriasis, ankylosing spondylitis, inflammatory bowel disease, and Reiter's syndrome.

Type I diabetes is associated with both HLA-DR3 and HLA-DR4 (choice D).

Rheumatoid arthritis is associated with HLA-DR4 (choice E).

A 47-year-old male presents with declining renal function characterized by oliguria, elevated blood urea nitrogen and creatinine, and hematuria. He also complains of nasal congestion and epistaxis. Review of systems is notable for occasional cough and hemoptysis. Examination shows mucosal ulceration and nasal septal perforation, but no polyps. Which of the following serum markers would likely be present in this case?

A. Anti-centromere antibody

B. Anti-Ro

C. Anti-SS-B

D. c-ANCA (cytoplasmic antinuclear cytoplasmic antibody)

E. Decreased erythrocyte sedimentation rate (ESR)

Explanation:

The correct answer is D. This patient has Wegener's granulomatosis, which is characterized by renal involvement, severe upper respiratory tract symptoms, and pulmonary involvement. Other organ systems may also be involved. The renal syndrome is a crescentic rapidly progressive glomerulonephritis leading to renal failure. The upper respiratory tract findings include sinus pain and drainage, and purulent or bloody nasal discharge with or without nasal ulcerations. Nasal septal perforation may follow. Pulmonary involvement may be clinically silent with only infiltrates present on x-ray, or it may present as cough and hemoptysis. c-ANCA is a marker for Wegener's granulomatosis, present in a high percentage of patients.

Anti-centromere antibody (choice A) is associated with approximately 90% of cases with CREST syndrome (calcinosis, Raynaud's phenomenon, esophageal motility syndrome, sclerodactyly, and telangiectasia) which is also called limited scleroderma.

Anti-Ro (choice B) is also called anti-SS-A and is associated with Sjögren's syndrome (70-95%).

Anti-SS-B (choice C) is associated with Sjögren's syndrome (60-90%).

Decreased ESR (choice E) is not a marker of Wegener's. Instead, a markedly elevated ESR is seen.

Additionally, mild anemia, thrombocytosis, leukocytosis, mild hypergammaglobulinemia (IgA), and mildly elevated rheumatoid factor are seen in this disorder.

A 57-year-old woman with a history of hypertension and arthritis is referred to a rheumatologist for evaluation. A complete blood count (CBC) is normal, and a mini-chem panel shows no electrolyte abnormalities. Her erythrocyte sedimentation rate (ESR) is elevated, and an antinuclear antibody test (ANA) is positive. Further antibody studies are performed, and the results are shown below.

Anti-histones high titer Anti-double stranded DNA not detected Anti-single stranded DNA not detected Anti-SSA not detected Anti-SSB not detected Anti-SCI-70 not detected Anti-Smith not detected Anit-centromere not detected Anti-RNP not detected

Which of the following diseases is suggested by these results?

A. CREST syndrome

B. Diffuse form of scleroderma

C. Drug-induced lupus

D. Sjögren's syndrome

E. Systemic lupus erythematosus

Explanation:

The correct answer is C. The single finding of high autoantibody titers to histones, without any other autoantibodies, is characteristic of drug-induced lupus. The most commonly implicated drugs are procainamide, hydralazine (given for hypertension), and isoniazid. Patients typically have milder disease than in SLE, and tend to have arthritis, pleuro-pericardial involvement, and, less commonly, rash. CNS and renal disease are not usually observed.

CREST syndrome (choice A) is a milder variant of scleroderma characterized by calcinosis, Raynaud's phenomenon, esophageal dysmotility, sclerodactyly and telangiectasia. Anti-centromere antibodies are diagnostic.

The diffuse form of scleroderma (choice B), also known as systemic sclerosis, causes fibrosis of the skin and internal viscera. This disorder is characterized by anti-SCI-70 and often low titers of many other autoantibodies.

Sjögren's syndrome (choice D) is characterized by dry eyes and dry mouth. Sjögren's syndrome in isolation is characteristically positive for anti-SSA and anti-SSB. If it accompanies rheumatoid arthritis, anti-RNP will be positive as well.

Systemic lupus erythematosus (choice E) is a multisystem disorder that is distinguished from drug-induced lupus by the presence of a wide variety of autoantibodies, including anti-double stranded DNA (anti-dsDNA).

A 24-year-old woman with a history of allergic rhinitis is involved in an automobile accident and sustains a splenic laceration. She undergoes abdominal surgery and is then transfused with four units of blood of the appropriate ABO and Rh type. As the transfusion progresses, she becomes rapidly hypotensive and develops airway edema, consistent with anaphylaxis. Which of the following pre-existing conditions best accounts for these symptoms?

A. AIDS

B. C1 esterase inhibitor deficiency

C. DiGeorge syndrome

D. Selective IgA deficiency

E. Wiskott-Aldrich syndrome

Explanation:

The correct answer is D. Patients with selective IgA deficiency may have circulating antibodies to IgA. Fatal anaphylaxis may ensue if they are transfused with blood products with serum containing IgA, although many patients with selective IgA deficiency are asymptomatic and never diagnosed. Symptomatic patients may have recurrent sinopulmonary infections and diarrhea, and also have an increased incidence of autoimmune and allergic diseases.

AIDS (choice A) predisposes for infections and neoplasms, but not anaphylaxis.

C1 esterase inhibitor deficiency (choice B) is an autosomal dominant disease characterized by recurrent attacks of colic and episodes of laryngeal edema, without pruritus or urticarial lesions. This disorder is also known as hereditary angioedema.

DiGeorge syndrome (choice C) is characterized by thymic aplasia and sometimes, hypoparathyroidism. The disorder is due to abnormal development of the third and fourth pharyngeal pouches. Wiskott-Aldrich syndrome (choice E) is a form of immunodeficiency associated with thrombocytopenia and eczema.

An 8-month-old boy baby is evaluated because of repeated episodes of pneumococcal pneumonia. Serum studies demonstrate very low levels of IgM, IgG, and IgA. This patient's condition is thought to be related to a deficiency of which of the following proteins?

A. Adenosine deaminase

B. Class III MHC gene

C. Gamma chain of the IL-2 receptor

D. Purine nucleotide phosphorylase

E. Tyrosine kinase

Explanation:

The correct answer is E. The patient in the question stem has X-linked (Bruton's) agammaglobulinemia, which is now thought to be due to a deficiency in a tyrosine kinase, leading to a B cell maturation arrest at the pre-B cell level.

Selective IgA deficiency has been linked to defective class III MHC genes (choice B).

Severe combined immunodeficiency is apparently a heterogeneous disease, and different subgroups have been linked to abnormalities of adenosine deaminase (choice A), the gamma chain of the IL-2 receptor (choice

C), and purine nucleotide phosphorylase (choice D).

An 18-year-old high school senior presents to her doctor with tender lymph nodes in her neck on the left side.

She has no significant past medical history. Two weeks ago, she updated her vaccines in preparation for college.

A lymph node biopsy is performed, which shows benign paracortical expansion and scattered multinucleated giant cells with eosinophilic cytoplasmic and nuclear inclusion bodies. Which of the following vaccines is most likely responsible for this woman's lymphadenitis?

A. Hepatitis B

B. Measles

C. Rubella

D. Smallpox

E. Tetanus

Explanation:

The correct answer is B. The Warthin-Finkeldey (WF) giant cell described above is pathognomonic for measles or the live attenuated measles vaccine. Most giant cells are composed of histiocytes, but the WF giant cell is created by fusion of lymphocytes. Although postvaccinal lymphadenitis may be seen with different vaccines, the usual reaction is immunoblastic proliferation within the paracortical regions of a hyperplastic lymph node.

Hepatitis B (choice A), rubella (choice C), and tetanus (choice E) are rarely associated with postvaccinial lymphadenitis.

Smallpox (choice D) is classically followed by tender regional adenopathy, one to several weeks following immunization. There are no associated giant cells.

A 21-year-old college student from Connecticut with a past history of Lyme disease presents with chronic pain and swelling in his right knee. He states that he has had problems with the knee for the past two years. Which of the following HLA alleles would you expect to be present in this individual?

A. HLA-B9

B. HLA-B17

C. HLA-B27

D. HLA-DR3

E. HLA-DR4

Explanation:

The correct answer is E. Approximately 60% of patients in the United States who contract Lyme disease, but are not treated with antibiotics, will develop frank arthritis. The pattern typically consists of intermittent attacks of oligoarticular arthritis in large joints (especially knees) lasting for weeks to months in a given joint. Patients with persistent arthritis have a higher frequently of HLA-DR4 class II MHC complex than patients with brief Lyme arthritis or normal controls. HLA-B9 (choice A), and HLA-B17 (choice B) are not thought to be associated with Lyme disease arthritis.

HLA-DR3 (choice D) is associated with a variety of disorders, but not Lyme disease arthritis.

HLA-B27 (choice C), although associated with reactive arthritis, is not associated with Lyme disease arthritis.

A 7-year-old girl is walking across a vacant lot and steps on a nail. The next day, her foot is sore and the wound appears inflamed. During these early stages of infection, which of the following compounds exert the most powerful chemotactic effect on neutrophils, causing them to migrate into the inflamed area?

A. C5a and IL-8

B. IL-1 and tumor necrosis factor

C. LTC4 and LTD4

D. PGI2 and PGD2

E. Thromboxane and platelet activating factor

Explanation:

The correct answer is A. The most important chemotactic factors for neutrophils are the complement factor C5a and the interleukin IL-8.

The cytokines IL-1 and tumor necrosis factor (choice B) have complex, similar actions, including stimulation of production of many acute-phase reactions, stimulation of fibroblasts, and stimulation of endothelium.

Leukotrienes LTC4 and LTD4(choice C) cause increased vascular permeability.

Prostaglandins PGI2 and PGD2(choice D) mediate vasodilation and pain.

Thromboxane and platelet activating factor (choice E) induce platelet changes.

One year after orthotopic liver transplantation for hepatitis C and cirrhosis, a 53-year-old man develops rising transaminase and bilirubin levels. In order to minimize chronic rejection injury to hepatic endothelial cells, immunosuppressive therapy is aimed at down-regulating which of the following components of the immune response?

A. Autoantibody production

B. Complement protein synthesis

C. HLA antigen expression

D. Mast cell degranulation

E. T-lymphocyte activity

Explanation:

The correct answer is E. Chronic rejection of any solid organ entails cellular injury to endothelial cells, resulting in intimal proliferation, fibrosis, and eventually ischemic injury to the graft. Immunosuppressive therapy is directed at controlling lymphocyte activity and minimizing cellular rejection.

Autoantibodies (choice A) are not involved in organ transplant rejection. The antibodies produced are alloantibodies directed only to the graft, but not to the host.

Complement proteins (choice B) are involved in the humoral component of acute rejection, and complement binding to alloantibodies increases graft damage. Complement protein production, however, is not affected by immunosuppressive therapy.

HLA antigen expression (choice C) is central to recognition of foreign cells in grafted tissue. HLA antigens are expressed constitutively by all normal cells, and immunosuppression does not affect their production.

Mast cell degranulation (choice D) is a component of the anaphylactic response (Type I hypersensitivity). Graft rejection is a Type IV hypersensitivity response, and does not involve mast cell degranulation.

A 40-year-old woman with systemic sclerosis presents to her physician with malaise. Physical examination is remarkable for jaundice, and serum chemistry studies demonstrate moderately elevated serum alkaline phosphatase, while aspartate aminotransferase (AST) and alanine aminotransferase (ALT) are only minimally elevated. Presence of which of the following autoantibodies would be most helpful in elucidating the likely etiology of the woman's liver disease?

A. Anti-double-stranded DNA

B. Anti-mitochondrial antibodies

C. Anti-phospholipid antibodies

D. Anti-self IgG

E. Anti-smooth muscle antibodies

Explanation:

The correct answer is B. Primary biliary cirrhosis is strongly associated with systemic sclerosis (scleroderma). This disorder characteristically affects intrahepatic bile ducts more than the hepatic parenchyma, at least in the earlier stages, and consequently causes a disproportionate increase in serum alkaline phosphatase compared to AST and ALT. The most distinctive markers for primary biliary cirrhosis are anti-mitochondrial antibodies, especially the M2 subtype. Primary biliary cirrhosis is also associated with a variety of other diseases, such as Sjögren's syndrome, rheumatoid arthritis, thyroiditis, celiac disease, and glomerulonephritis.

Anti-double-stranded DNA (choice A) and anti-phospholipid antibodies (choice C) are markers for systemic lupus erythematosus.

Anti-self IgG (choice D; also known as rheumatoid factor) is a marker for rheumatoid arthritis.

Anti-smooth muscle antibodies (choice E) are seen in autoimmune hepatitis, which is not as strongly associated with scleroderma as primary biliary cirrhosis.

Which of the following events occurs first in the differentiation sequence of human B cells in the bone marrow?

A. Cytoplasmic mu chains present in the B cell

B. Immunoglobulin heavy chain rearrangement

C. Immunoglobulin light chain rearrangement

D. Surface IgD and IgM present on the B cell

E. Surface IgM present on the B cell

Explanation:

The correct answer is B. The first event that occurs in the pre-B cell (progenitor) is the gene rearrangement of the heavy chain. The D gene and J gene recombination event occurs first, followed by V recombination with the D-J region.

The cytoplasmic mu chains (choice A) are the result of immunoglobulin heavy chain rearrangement, the production of the messenger RNA from this rearrangement, and the ribosomal synthesis of the protein mu chain. This is the second event that occurs in the B-cell maturational sequence.

Light chain rearrangement (choice C) occurs when recombination events occur with the V gene and J gene from the light chain germ line. After this recombination has occurred, and the messenger RNA for this germ line has produced the light chain protein, the light chains and heavy chains form.

The complete IgM molecule and IgD molecules (choice D) are present on the surface of only the mature B cells. This event is the last event to occur during the differentiation and development of B cells in the bone marrow. The complete IgM molecule (choice E) is present on the surface of the immature B cell. This event is one of the last events to occur in the differentiation and development of B cells.

A 4-year-old boy is seen by his pediatrician for epistaxis. The patient has a history of multiple bacterial and viral respiratory tract infections and eczema. An uncle had similar problems. Physical examination is remarkable for multiple petechial lesions on the skin and mucous membranes. Serum IgE is increased, and platelets are decreased. Which of the following is the most likely diagnosis?

A. Acquired hypogammaglobulinemia

B. Ataxia telangiectasia

C. DiGeorge syndrome

D. Selective IgA deficiency

E. Wiskott-Aldrich syndrome

Explanation:

The correct answer is E. Wiskott-Aldrich syndrome is an X-linked condition characterized by eczema, thrombocytopenia, and repeated infections. Affected children may present with bleeding and often succumb to complications of bleeding, infection, or lymphoreticular malignancy. The platelets are small, have a shortened half-life, and appear to be deficient in surface sialophorin (CD43). Splenectomy can correct the thrombocytopenia, but not the immune defect. Serum IgM is usually decreased, while IgE is frequently increased. Mutations in the Wiskott-Aldrich serum protein (WASP) gene on the short arm of the X chromosome are responsible for this disease.

Acquired hypogammaglobulinemia (choice A) is a disease of adults characterized by normal numbers of B cells but low immunoglobulin production.

Ataxia telangiectasia (choice B) is an autosomal recessive disease characterized by progressive cerebellar dysfunction, telangiectasias, and a variable immunodeficiency.

DiGeorge syndrome (choice C) is a developmental malformation leading to thymic aplasia and, sometimes, hypoparathyroidism.

Selective IgA deficiency (choice D) is a relatively common condition characterized by low levels of IgA.

A 34-year-old woman presents with fatigue, malaise, and swollen, tender joints. Physical examination is significant for a maculopapular eruption over sun-exposed areas, including the face. Examination of a peripheral blood smear reveals mild thrombocytopenia. Which of the following autoantibodies, if present, would be most specific for the diagnosis of the patient's disorder?

A. Anti-centromere antibody

B. Anti-IgG antibody

C. Antinuclear antibody

D. Anti-Sm (Smith antigen) antibody

E. Anti-SS-A (Ro) antibody

Explanation:

The correct answer is D. The patient described probably has systemic lupus erythematosus (SLE). This is a systemic disorder that often presents with fatigue, malaise, fever, gastrointestinal symptoms, arthralgias, and myalgias. Hematologic abnormalities include anemia of chronic disease, hemolytic anemia, leukopenia, lymphocytopenia, and thrombocytopenia. A circulating anticoagulant may prolong the APTT (activated partial thromboplastin time). Cutaneous manifestations include a malar rash and a generalized maculopapular eruption, both of which are photosensitive. Antibodies to the Smith antigen (core proteins of small ribonucleoproteins found in the nucleus) are present in only 20-30% of patients with SLE, but are quite specific for the disease, occurring only rarely in other autoimmune diseases.

Anti-centromere antibody (choice A) is specific for the CREST (Calcinosis, Raynaud's syndrome, Esophageal dysfunction, Sclerodactyly, and Telangiectasia) variant of progressive systemic sclerosis (scleroderma).

Rheumatoid factor is actually an autoantibody directed against the Fc portion of the IgG molecule (choice B). It is found in more than two-thirds of patients with rheumatoid arthritis.

The majority of patients with SLE (around 95%) develop antinuclear antibodies (ANA; choice C), so this test is quite sensitive, but not very specific for SLE. ANA occur in other inflammatory disorders, autoimmune diseases, viral diseases, and in a number of normal individuals. Antibodies to double-stranded DNA are more specific for SLE, but are not included as an answer choice.

Anti-SS-A antigen (choice E) refers to antibodies to certain ribonucleoproteins, which are fairly specific for Sjögren's syndrome.

A four-year-old boy is brought to the pediatrician because of several "boils" on his arm. His mother tells the physician that the boy has had similar lesions on several previous occasions that were treated successfully with antibiotics. She denies any history of eczema or typical childhood illnesses such as measles or chicken pox. The child has had all of his immunizations. Laboratory examination reveals a normal complete blood count, immunoglobulin levels, B cell and T cell counts, and complement levels. Serum calcium and parathyroid hormone levels are also normal. The nitroblue tetrazolium test is negative. Which of the following diagnoses is most consistent with these data?

A. Bruton's agammaglobulinemia

B. Chronic granulomatous disease

C. DiGeorge syndrome

D. SCID (severe combined immunodeficiency disease)

E. Wiskott-Aldrich syndrome

Explanation:

The correct answer is B. The fact that the boy had several different infections with pyogenic bacteria requiring antibiotics suggests an inability of phagocytes to kill bacteria. The nitroblue tetrazolium test (NBT) for reactive oxygen intermediates was negative, indicating that the boy suffers from chronic granulomatous disease (CGD). CGD is most often due to a defect in NADPH oxidase, which is necessary for leukocyte hydrogen peroxide production.

In Bruton's agammaglobulinemia (choice A), patients have very low levels of circulating immunoglobulins. There is a virtual absence of B cells, but pre-B cells are present. These patients also experience frequent pyogenic bacterial infections, but the boy in the question had normal immunoglobulin levels and normal B cell counts. DiGeorge syndrome (choice C) results when the 3rd and 4th pharyngeal pouches fail to develop in normal fashion and the individual is missing the thymus gland and parathyroid glands. Hypocalcemia, low parathyroid levels and a T cell abnormality are typical.

In SCID (severe combined immunodeficiency disease) (choice D) both B cells and T cells may be absent, or if present, may not function properly. In this patient, the B and T cell counts were normal. The immunoglobulin levels were normal, so the B cells were functioning, and the patient was successfully immunized, so his T cells were functioning.

Wiskott-Aldrich syndrome (choice E), an immune deficiency disease that develops in the first year of life, is characterized by pyogenic infections, eczema, and thrombocytopenia.

A 45-year-old man presents to a physician with complaints of double vision and ptosis. The patient has noticed that these problems are minor in the early morning, but become progressively more severe during the course of the day. Symptoms markedly improve after a test dose of edrophonium. This condition is usually related to autoantibodies directed against which of the following?

A. Acetylcholine receptor

B. Double-stranded DNA

C. Neutrophil cytoplasmic proteins

D. SS-A (Ro)

E. TSH receptor

Explanation:

The correct answer is A. The disease is myasthenia gravis, which is the result of autoantibodies directed against the acetylcholine receptor at the neuromuscular junction. Many patients with myasthenia gravis have related thymic hyperplasia or thymoma.

Antibodies to double-stranded DNA (choice B) are a feature of systemic lupus erythematosus, a collagen-vascular disease.

Antibodies to neutrophil cytoplasmic proteins (choice C), called antineutrophil cytoplasmic autoantibodies (ANCA), are a feature of Wegener's granulomatosis.

Antibodies to SS-A (choice D) are a feature of Sjögren's syndrome, characterized by autoimmune inflammation of the lacrimal and salivary glands

Antibodies to the TSH receptor (choice E) are a feature of Graves' disease. Stimulation of the receptor by the autoantibodies is responsible for producing a hyperthyroid state.

A child stepped on a piece of glass 1 day ago, and an active inflammatory reaction is occurring in her wound, with large numbers of neutrophils attracted to the inflammation site. Which of the following is the major chemotactic factor responsible for attracting neutrophils?

A. C3b

B. C5a

C. IgM

D. IL-2

E. Lysozyme

Explanation:

The correct answer is B. In active inflammation, the complement system has been activated and C5a is being produced. C5a is a strong chemoattractant to neutrophils and other phagocytic cells.

C3b (choice A) is an excellent opsonin of pathogenic organisms; when an organism is coated with C3b, it is more easily phagocytized. C3b is formed via the classic and alternative complement pathways. IgM (choice C) is the first immunoglobulin produced in the primary immune response. IgM cannot cross the placenta, but it is a powerful activator of complement; elevated levels in the newborn are associated with an acute infection with a pathogen.

IL-2 (choice D) is a powerful interleukin that stimulates T helper 1 cells. It also stimulates natural killer cells and T cytotoxic CD8 lymphocytes, but is not chemotactic for neutrophils.

Lysozyme (choice E) is a material present in tears, mucous, vaginal secretions, and other body fluids. It is active against the peptidoglycan of bacterial cell walls, splitting the backbone structure of the peptidoglycan (N-acetylglucosamine and N-acetyl muramic acid polymers).

A 65-year-old woman is evaluated for symmetrical swelling of the proximal phalangeal joints. Physical examination also reveals large subcutaneous nodules over the extensor surfaces of both arms. Autoantibodies directed against which of the following antigens would most likely be demonstrated by serum studies?

A. Acetylcholine receptor

B. Double stranded DNA

C. Histones

D. IgG

E. Ribonucleoprotein

Explanation:

The correct answer is D. The disease is rheumatoid arthritis, and the autoantibody is rheumatoid factor, which is usually an IgM or IgG (or less commonly IgA) directed against the constant region of autologous IgG.

Autoantibody directed against acetylcholine receptor (choice A) is a feature of myasthenia gravis.

Autoantibody directed against double stranded DNA (choice B) is a feature of systemic lupus erythematosus.

Autoantibody directed against histones (choice C) is a feature of drug-induced lupus.

Autoantibody directed against ribonucleoprotein (choice E) is a feature of mixed connective tissue disease.

Which of the following is the most important costimulatory signal provided to a T cell from an antigen-presenting cell?

A. B7 molecules interacting with CD 28

B. B7 molecules interacting with LFA- 1

C. ICAM-I interacting with LFA-1

D. LFA-3 interacting with CD 28

E. MHC class II interacting with T cell receptor

Explanation:

The correct answer is A. The B7 molecule on the cell surface of the antigen-presenting cell reacts with the CD 28 molecule on the T cell surface for maximal costimulatory signals.

The B7 molecule on the surface of the antigen presenting cell reacts only with CD 28 and does not react with LFA-1 (choice B) adhesion molecule.

The ICAM- I on the surface of an antigen presenting cell reacts with the LFA- I (choice C) on the surface of a T cell for the purpose of cell-to-cell adhesion and does not function for costimulation.

The LFA-3 (CD58) is an adhesion molecule on the surface of an antigen presenting cell. It does not react with a CD28 (choice D) costimulatory molecule on the T cell surface.

The MHC class II molecule with its epitope does interact with a specific T cell receptor (TCR) (choice E), but this is not termed costimulatory. However, the interaction does stimulate the T cell to produce interleukins for further cell division.

Cytotoxic T cells induced by infection with virus A will kill target cells

A. from the same host infected with any virus

B. infected by virus A and identical at class I MHC loci to the cytotoxic T cells

C. infected by virus A and identical at class II MHC loci to the cytotoxic T cells

D. infected with any virus and identical at class I MHC loci to the cytotoxic cells

E. infected with any virus and identical at class II MHC loci to the cytotoxic cells

Explanation:

The correct answer is B. The CD8+ cytotoxic T cells have antigen specific T-cell receptors (TCR) on their membranes that will recognize and bind to self class I antigens. Since the viral peptides are presented as a complex with the self class I antigens, the CD8+ cells can now recognize and react to the virus A peptides. Remember that class I antigens are expressed on all nucleated cells and platelets.

Which of the following genes involved in the synthesis of immunoglobulins are linked on a single chromosome?

A. C gene for gamma chain and C gene for alpha chain

B. C gene for gamma chain and C gene for kappa chain

C. V gene for kappa chain and C gene for the epsilon chain

D. V gene for lambda chain and C gene for kappa chain

E. V gene for lambda chain and V gene for heavy chain

Explanation:

The correct answer is A. The genes for the synthesis of the entire heavy chain are present on human chromosome 14. Of the options given above, the only genes that are present on one chromosome are for the C (constant) regions of the heavy chains. Remember, the heavy chains determine the identity of the immunoglobulin isotypes: IgG, IgM, IgA, IgD, and IgE.

Choice B: The C gene for the gamma heavy chain is on chromosome 14 and the C gene for the kappa light chain gene is on chromosome 2.

Choice C: The V gene for the kappa light chain gene is on chromosome 2. The C gene for the epsilon heavy chain is on chromosome 14.

Choice D: The V gene for the lambda light chain gene is on chromosome 22. The C gene for the kappa light chain gene is on chromosome 2.

Choice E: The V gene for the lambda light chain gene is on chromosome 22 and the V gene for the heavy chain gene is on chromosome 14.

What is the role of the macrophage during antibody formation?

A. Activation of cytotoxic CD8 T cells

B. Delayed hypersensitivity reaction

C. Lysis of virus-infected cells

D. Processing antigen and presenting it to T helper CD4 cells

E. Synthesis of immunoglobulin

Explanation:

The correct answer is D. The macrophage phagocytizes exogenous antigens (for example a bacterium), degrading the antigen into small epitopes and presenting them, on MHC class II molecules on its surface, to CD4 T helper cells.

Macrophages do not activate cytotoxic CD 8 T lymphocytes (choice A). The major activator of cytotoxic CD 8 T lymphocytes is IL-2 from CD4+ THl cells.

Delayed hypersensitivity reactions (choice B) are the results of CD4+ THl cells. These cells do not produce antibody. They secrete gamma interferon and interleukin 2 (IL-2), stimulating more cells to become involved in the delayed hypersensitivity reaction.

The cells that participate in lysis of virus infected cells (choice C) are cytotoxic CD8+ T lymphocytes that react with MHC class I molecules containing epitopes of the virus from the infected cell. Macrophages do not participate in this activity.

Macrophages never synthesize antibody (choice E). B cells initially produce antibody, then are converted to plasma cells or memory B cells. The stimulus for the production of this antibody comes from T helper cells that were stimulated by epitopes presented to them by macrophages.

A 60-year-old female presents with progressive tightening and hardening of the skin on her arms and face that has recently caused disfigurement and difficulty performing manual activities. You suspect an autoimmune disorder, and order an antinuclear antibody panel. The results show antitopoisomerase I and no anticentromere antibodies. You advise the patient that, if left untreated, her disease will most likely A. convert to a dermatologic malignancy B. progress to involve other skin surfaces C. progress to potentially fatal systemic fibrosis D. remit completely E. stabilize, with residual dermal fibrosis Explanation: The correct answer is C. This patient has scleroderma (or systemic sclerosis), an autoimmune connective tissue disorder. The skin is most frequently involved in this disease and is characterized by excessive tissue fibrosis. There is evidence for both an immunologic and vascular etiology to the disease. Almost all patients with scleroderma have antinuclear antibodies. Those with the antitopoisomerase antibody usually develop diffuse systemic sclerosis, and they usually die from consequences of systemic disease such as pulmonary fibrosis or malignant hypertension. Scleroderma does not predispose to dermatologic malignancies (choice A). Progression to involve other skin surfaces (choice B) without visceral involvement suggests limited systemic sclerosis, associated with an anticentromere antibody. 96% of patients with another limited form of systemic sclerosis, the CREST syndrome (calcinosis, Raynaud's phenomenon, esophageal dysmotility, sclerodactyly, telangiectasias), also have an anticentromere antibody. Scleroderma is a slowly progressive disease that will not stop or spontaneously reverse on its own (choices D and E). A 60-year-old alcoholic male with a long history of cigarette smoking is brought to the emergency department after being found behind the neighborhood bar at 4 AM in freezing weather. On arrival, he is lethargic and experiences a shaking chill. His heart rate is 106, his breathing is labored with diffuse rales, and his temperature is 102.5 degrees Fahrenheit. His sputum is blood tinged, containing numerous gram-positive cocci which are identified as Streptococcus pneumoniae. The man is treated with penicillin and his condition improves over the next few days. Which of the following immune effector mechanisms was most important in completely clearing this infection? A. ADCC (Antibody dependent cell cytotoxicity) B. Complement mediated opsonization C. Cytotoxic T cell lymphocytes D. LAK cells E. Natural killer cells Explanation: The correct answer is B. One of the most efficient mechanisms for eliminating extracellular pathogenic bacteria is by opsonization and phagocytosis by macrophages. The IgG and IgM antibody produced in response to the organism reacts with the capsular structure, stimulating the activation of the classical pathway of the complement system. This pathway produces large amounts of C3b that coat the organism, preparing it for phagocytosis. ADCC cells (choice A) are actually natural killer (NK) cells that find virally infected cells and tumor cells that have been coated with antibody and react with and destroy them. These cells do not destroy antibody-coated bacteria, only body cells that are coated with antibody. Cytotoxic T lymphocytes (choice C) react only with cells that have antigen epitopes presented in association with class I MHC molecules. An example would be a virus epitope from a virally infected cell presented by class I molecules on the surface of the cell. LAK cells (choice D) are NK cells that have been activated by IL-2. They are considered to be superactivated NK cells. The NK cells (choice E) are cells of the innate immune system that destroy virally infected cells or tumor cells. This does not involve antibody and it does not involve extracellular pathogens. A small 9-month-old male with a history of recurrent pyogenic infections is seen in a clinic. Immunoglobulin levels and a CBC are performed. The CBC is normal except for slight neutropenia and thrombocytopenia. Determination of immunoglobulin levels indicates elevated IgM, but deficiencies of IgG and IgA. The underlying defect involves which of the following molecules? A. CD40 ligand (CD40L) on the T cell B. CD40 molecule on the B cell C. Gamma interferon D. Interleukin-2 (IL-2) E. Interleukin-3 (IL-3) Explanation: The correct answer is A. Patient's with hyper-IgM syndrome (HIGM) experience very little, if any, isotype switching. The B cells in these patients cannot undergo the switch from IgM to IgG, IgA, or IgE that normally occurs during B-cell maturation. When B cells undergo isotype switching they require two factors: IL-4, which binds to a specific receptor on the B cell, and the CD40 molecule on the B-cell surface, which binds to the CD40 ligand (CD40L) on an activated T-cell surface. The deficiency is due to mutations in the CD40L. This immunodeficiency results in patients who are IgG- and IgA-deficient, but synthesize large amounts of polyclonal IgM. Affected individuals are susceptible to pyogenic infections, and often form IgM autoantibodies to neutrophils, platelets, or tissue antigens. The disease is inherited as an X-linked recessive in 70% of the cases. The problem is due to mutations in the CD40L, not the CD40 molecule on the B cell (choice B). Gamma-interferon (choice C) is primarily a type I helper T cell (TH1) cytokine, although it does inhibit the TH2 lineage response to specific antigens. A decrease in IL-2 (choice D) would inhibit the cell-mediated immune response and the patient would be susceptible to infection with intracellular microorganisms. IL-3 (choice E) is considered a growth factor for hematopoietic stem cells and mast cells. The patient had a normal CBC except for slight neutropenia and thrombocytopenia. A deficiency of the complement protein C4 would inhibit which of the following complement activities? A. Completion of the classic pathway to the splitting of C3 B. Formation of C3b for opsonization C. Formation of C5 convertase via the alternative pathway D. Formation of C5a for chemotactic attractant for neutrophils E. Formation of the membrane attack complex Explanation: The correct answer is A. The classic complement pathway involves C1, C4, and C2 to the point of splitting C3. C3 is then split to yield C3b and C3a. A deficiency of C4 would have no effect on any of the other answer choices listed. The complement protein C3 can be split into C3a and C3b (choice B) using the alternative pathway. The additional proteins required in this pathway would be factors B and D and properdin. The C5 convertase enzyme can be formed in the alternative pathway (choice C) without using C4. The C5 molecule could be split into C5a (choice D) and C5b using the alternative pathway, without the involvement of C4. The membrane attack complex (choice E; C5b, C6, C7, C8, and polymers of C9) lyses the pathogenic cell. This process does not require the alternative pathway and would not require C4. Anti-centromere High titer Anti-Scl-70 Not detected Anti-Pol-I Not detected Anti-RNP Not detected Anti-dsDNA Not detected Which of the listed diseases is most strongly suggested by the above antibody studies? A. CREST B. Diffuse scleroderma C. Mixed connective tissue disease D. Sjögren's syndrome E. Systemic lupus erythematosus Explanation: The correct answer is A. Anti-centromere antibodies are a marker for the CREST (calcinosis, Raynaud's phenomenon, esophageal dysfunction, sclerodactyly, and telangiectasias) variant of scleroderma. The markers for diffuse scleroderma (choice B), anti-Scl-70 and anti-Pol-I, are usually negative in CREST syndrome, as are most other autoantibodies. The marker for mixed connective tissue disease (choice C) is anti-RNP. The markers for Sjögren's syndrome (choice D) are anti-SSA, anti-SSB, and anti-RAP. Systemic lupus erythematosus (choice E) typically produces a large number of autoantibodies, of which anti-dsDNA and anti-Sm are the most distinctive. What is the role of class II MHC proteins on donor cells in the process of graft rejection? A. They are recognized by helper T cells, which then activate cytotoxic T cells to kill the donor cells B. They are the receptors for interleukin-2, which is produced by macrophages when they attack the donor cells C. They cause the release of perforins to lyse the cells D. They induce IgE, which mediates graft rejection E. They induce the production of blocking antibodies that protect the graft Explanation: The correct answer is A. Class II MHC proteins are expressed on the surfaces of macrophages, dendritic cells, and B cells; this complex of molecules is recognized by CD 4+ helper T cells. The T cells of the transplant recipient recognize allogeneic MHC molecules on the surface of an antigen-presenting cell of the donor. It is thought that interstitial dendritic cells of the donor are the most important immunogens because not only do they express class I and II HLA molecules, but they are also endowed with co-stimulatory molecules. CD 8+ cytotoxic T cells recognize the class I molecules. CD 4+ cells proliferate as Th1 cells and produce interleukin 2, which causes differentiation of the CD 8 cells. The CD 8+ cytotoxic cells of the recipient then cause lysis of the donor cells. Interleukin-2 (choice B) activates T cells by binding to high-affinity IL-2 receptors (IL-2R). Perforins (choice C) are produced by CD 8+ cytotoxic lymphocytes as they bind to Class I MHC molecules. The perforins damage the donor cell membranes, resulting in lysis. IgE-mediated reactions (choice D) are not associated with graft rejection. Blocking antibodies (choice E) are employed as a form of immunosuppressive therapy. Antilymphocyte globulins and monoclonal anti-T cell antibodies (monoclonal anti-CD3) are used to inhibit rejection of the graft. This process does not involve class II MHC proteins on donor cells. A 33-year-old single mother of two young children visits her physician because of an oral ulcer. A review of systems is significant for fatigue, myalgia, and joint pain. Laboratory results demonstrate leukopenia, and a high-titered antinuclear antibody. A speckled staining pattern due to anti-Sm is seen with immunofluorescence; urinary protein is elevated. Which of the following is the most likely diagnosis? A. Generalized fatigue B. Goodpasture's syndrome C. Mixed connective tissue disease D. Scleroderma E. Systemic lupus erythematosus Explanation: The correct answer is E. Systemic lupus erythematosus (SLE) is a prototype connective tissue disease. The diagnosis requires four criteria to be met from a list of eleven possible criteria: malar rash, discoid rash, photosensitivity, oral ulcers, arthritis, serositis, renal disorder, neurologic disorder, hematologic disorder, immunologic disorder, and antinuclear antibody. This patient also has anti-Sm, which is pathognomonic for SLE, but is only found in 30% of the affected patients. Antinuclear antibodies (ANA) are present in 95-100% of cases of SLE; anti-double-stranded DNA is found in 70% of the cases. Generalized fatigue (choice A) due to being a single working mother of two children could well be a possibility, but the presence of the other criteria make SLE more likely. Goodpasture's syndrome (choice B) is characterized by linear disposition of immunoglobulin, and often C3, along the glomerular basement membrane (GBM). Glomerulonephritis, pulmonary hemorrhage, and occasionally idiopathic pulmonary hemosiderosis occur. Mixed connective tissue disease (choice C) is an overlap syndrome characterized by a combination of clinical features similar to those of SLE, scleroderma, polymyositis, and rheumatoid arthritis. These patients generally have a positive ANA in virtually 100% of the cases. High titer anti-ribonucleoprotein (RNP) antibodies may be present, generating a speckled ANA pattern. Anti-RNP is not pathognomonic for mixed connective tissue disease, since it can be found in low titers in 30% of the patients with SLE. Scleroderma (choice D) is characterized by thickening of the skin caused by swelling and thickening of fibrous tissue, with eventual atrophy of the epidermis. ANA are often associated with the disease, but the staining pattern is generally nucleolar. A superantigen is a bacterial product that A. binds to B7 and CD28 costimulatory molecules B. binds to the β chain of TCR and MHC class II molecules of APC stimulating T cell activation C. binds to the CD4 + molecule causing T cell activation D. is presented by macrophages to a larger-than-normal number of T helper CD4 + lymphocytes E. stimulates massive amounts of IgG synthesis because of its large size Explanation: The correct answer is B. A superantigen, such as TSST- 1 or staphylococcal enterotoxin, cross-links the variable domain of the TCR β chain to the MHC class II molecule and specifically induces massive T cell activation. The superantigen does not bind the B7 and CD28 costimulatory molecules (choice A). Instead, the costimulatory molecules bind to each other to stimulate the reaction between the antigen-presenting cell and T cell. The superantigen does not bind the CD 4 molecules (choice C) but instead binds on the other side of the TCR receptor complex. The term superantigen has nothing to do with the antigen being presented by macrophages to T cells (choice D). The term superantigen has nothing to do with its size or its ability to stimulate antibody production (choice E). The term superantigen is used because of its unusual ability to create massive T cell activation by the unique type of binding. The blood from an 8-year-old boy was analyzed by flow cytometry. The exact number of B cells was counted. Which of the following cell surface markers was likely used to identify the B cells in this blood sample? A. CD3 B. CD4 C. CD8 D. CD19 E. CD56 Explanation: The correct answer is D. The best markers for identification of B cells are CD19, CD20, and CD21. The CD21 marker is a receptor for EBV (Epstein-Barr Virus). The CD3 marker (choice A) is present on all T cells with either a CD4 or CD8 marker. This is the marker that is used to identify total T cell count in a blood sample. The CD3 marker is used for signal transduction in the different T cells. The CD4 marker (choice B) is used to identify T helper cells. These are the cells that recognize exogenous peptides presented on MHC class II molecules by macrophages. CD4+ T helper cells are also involved in cell-mediated delayed hypersensitivity, production of cytokines for stimulation of antibody production by B cells, and stimulation of macrophages. The CD8 marker (choice C) is used to identify cytotoxic T cells. These are the cells that recognize viral epitopes attached to the MHC class I molecules of a virally infected cell. The CD56 marker (choice E) is used to identify NK(natural killer) cells. These cells are important in innate host defense, specializing in killing virally infected cells and tumor cells by secreting granzymes and perforins. Which of the following is an example of a type II hypersensitivity? A. A patient with berylliosis B. A patient with heat intolerance, sinus tachycardia, and proptosis of the eyes C. Eczematous reaction on the dorsum of the foot in a patient who washed his socks in a new detergent D. Glomerulonephritis in a patient with systemic lupus erythematosus E. Wheal and flare reactions and vesicles at multiple sites on the lower legs in a patient attacked by fire ants Explanation: The correct answer is B. Type II hypersensitivity is mediated by antibodies directed toward antigens that are present on the surface of cells or other tissue components. The antigen may be intrinsic to the cell membrane or may take the form of an exogenous antigen that is adsorbed to the cell surface. The patient described in choice B has Graves disease, which is an autoimmune form of hyperthyroidism produced by autoantibodies directed against the TSH (thyroid stimulating hormone) receptor. These antibodies are called LATS (long-acting thyroid stimulator) and stimulate thyroid function, resulting in the release of thyroid hormones. In berylliosis (choice A), noncaseating granulomas typically are present in the lungs and hilar lymph nodes. This is a form of type IV hypersensitivity. An eczematous reaction (choice C) associated with washing clothes in a new detergent may either represent type IV hypersensitivity or a non-immune reaction associated with direct toxicity from some component of the soap penetrating the skin. Glomerulonephritis in systemic lupus erythematosus (choice D) is due to the deposition of antigens in the glomerular basement membrane with the resultant formation of antigen-antibody complexes. These complexes activate the complement cascade, which causes neutrophils to enter the area and produce tissue damage. Wheal and flare reactions (choice E) are cutaneous manifestations of type I hypersensitivity. T cells that have a low affinity for MHC class I molecules differentiate in the thymus to become which type of cell? A. CD 8 + cytotoxic lymphocyte B. Gamma-delta T cell C. Natural killer cell D. T helper 1 cell E. T helper 2 cell Explanation: The correct answer is A. CD8+ cytotoxic T lymphocytes are positively selected in the thymus because they have low affinity for MHC class I molecules. If they had possessed high affinity for the MHC class I molecules, they would have been eliminated (negative selection) because of the danger of autoimmune disease. Also, cells with no affinity for MHC class I molecules would be eliminated. The gamma-delta designation in a T cell (choice B) refers to type of receptor on the cell. Most T-cell receptors are alpha-beta receptors, but some are of a different isotype termed gamma-delta. Natural killer cells (choice C) are large granular lymphocytes that are part of the innate immune response. Natural killer cell function does not depend on MHC class I or class II molecules; it simply kills tumor cells or virally infected cells. T helper 1 (choice D) and T helper 2 cells (choice E) would have a low affinity for class II MHC molecules in order to survive in the thymus. If they had no affinity or strong affinity for these molecules, they would have been eliminated. A 36-year-old farmer has been exposed to poison ivy on several different occasions and usually develops very severe skin lesions. He enrolls in an immunological study at an urban medical center. A flow cytometric measurement of T cells reveals values within the normal range. An increased serum concentration of which of the following cytokines would decrease the likelihood of a delayed-type hypersensitivity reaction in this individual? A. Gamma interferon B. IL-2 C. IL-4 D. IL-8 E. IL-10 Explanation: The correct answer is E. The IL-10 cytokine is produced by T helper 2 (TH2) cells and inhibits T helper l (TH1) cells. Since the cytokines from TH1 cells stimulate cell-mediated immunity and delayed hypersensitivity, an increased level of IL-10 would decrease the likelihood of a delayed type hypersensitivity reaction. Other cytokines from TH2 cells stimulate B cells to produce antibody. The gamma interferon cytokine (choice A) from T helper l cells inhibits TH2 cells. It also activates NK cells and activates macrophages. The IL-2 cytokine (choice B) stimulates TH1 subset, CD8 T cytotoxic cells, and activates NK cells. It is one of the most active cytokines and is involved in many other reactions. The IL-4 cytokine (choice C) from TH2 cells stimulates B cells to produce antibody, inhibit macrophages, and stimulates class switching from IgG isotype to IgE isotype. The IL-8 cytokine (choice D) is produced by macrophages and is chemotactic for neutrophils. This cytokine is most important in stimulating an inflammatory reaction and attraction of neutrophils to the site. A 41-year-old patient informs her physician that in her childhood, she experienced two bouts of rheumatic fever. Although she appears to be well at present, which of the following sequelae of rheumatic fever is most likely to present as a chronic disease in her later years? A. Arthritis B. Fibrinous pericarditis C. Mitral valve disease D. Myocarditis E. Neurological disease Explanation: The correct answer is C. After an initial attack of rheumatic fever, an affected individual is at increased risk for developing recurrent rheumatic disease after each pharyngeal infection by beta-hemolytic streptococci. Any one of the manifestations of rheumatic disease can reoccur, but will usually resolve. In marked contrast, the damage to the heart valves caused by rheumatic fever can lead to fibrous scarring and deformity, producing valvular dysfunction (particularly mitral stenosis), which may lead to heart failure in late adulthood. The arthritis (choice A) produced by rheumatic fever is transitory, and resolves after the attack has run its course. There are no long-term complications of the arthritis. Fibrinous pericarditis (choice B), like almost any fibrinous inflammatory response, is an acute process. Although the patient may develop some scarring of the pericardium, this is generally not sufficient to produce long-lasting consequences. Myocardial involvement (choice D) by rheumatic fever is typically seen as Aschoff bodies, collections of fibrinoid necrosis, lymphocytes, plasma cells, and histiocytes within the myocardium. When the event resolves, the Aschoff bodies will be replaced by discrete fibrous scars. No appreciable myocardial dysfunction ensues. Sydenham's chorea, the neurologic manifestation of rheumatic fever, leaves no lasting neurological effects (choice E). A formula-fed 1-month-old boy is exposed to his sister, who has chickenpox. He does not develop signs of varicella. His mother had the infection 5 years ago. Which class of immunoglobulins did he acquire from his mother in utero that protected him from this virus? A. IgA B. IgD C. IgE D. IgG E. IgM Explanation: The correct answer is D. This baby is exhibiting passive immunity acquired from his mother in utero. IgG is the only class of immunoglobulins that can cross the placenta. As such, IgG molecules diffuse into the fetal circulation, providing immunity. This circulating maternal IgG protects the newborn during the first 4-6 months of life. Note that IgG is also capable of opsonization and complement activation (a feature shared with IgM). IgA (choice A) functions in the secretory immune response. The secretory form of this immunoglobulin (sIgA) is found in tears, colostrum, saliva, breast milk, and other secretions. It is produced by the plasma cells in the lamina propria of the GI and respiratory tracts. IgD (choice B) functions as a cell surface antigen receptor on undifferentiated B cells. IgE (choice C) is involved in the allergic response and immediate hypersensitivity reactions. The Fc region of IgE binds to the surface of basophils and mast cells. Antigen binding to two IgE molecules leads to mast cell degranulation and the release of leukotrienes, histamine, eosinophil chemotactic factors, and heparin. IgM (choice E) is the first antibody detected in serum after exposure to antigen. IgM circulates as a pentamer and thus has five Fc regions. This structure makes it especially effective in fixing complement. Isohemagglutinins, rheumatoid factors, and heterophile antibodies are all IgM. Administration of the DPT vaccine (diphtheria toxoid, pertussis products, and tetanus toxoid) would stimulate which of the following types of immunity? A. Adoptive B. Artificial active C. Artificial passive D. Natural active E. Natural passive Explanation: The correct answer is B. Administration of the DPT vaccine stimulates the innate immune system to produce antibody and memory cells against this mixture. Active immunity is when we produce our own antibody. Artificial refers to the fact that the stimulus was the vaccination with the antigens in question. Adoptive immunity (choice A) involves the patient receiving cells from another host who had been stimulated to produce their products. Artificial passive immunity (choice C) refers to the immunity produced by receiving an injection of antibody. An example is the administration of immune globulin directed against hepatitis A after an individual had been exposed to it. Active immunity (choice D) means that we are stimulated to produce our own antibodies. The term natural active refers to the fact that we received the stimulus (antigen) by a natural means, such as exposure to the organism. Natural passive immunity (choice E) refers to the immunity a fetus receives via the placenta. The term passive means that the fetus received the antibody from another source and did not make it itself. A full-term baby boy is delivered after an uneventful pregnancy, and is well for the first 2 years of his life. He receives all his immunizations without any complications. Starting around his 2nd birthday, the mother begins to note frequent upper respiratory tract infections, and the child is hospitalized three times for pneumonia. Laboratory testing would most likely reveal a deficiency of which of the following immunoglobulins in this child? A. IgA B. IgD C. IgG D. IgM Explanation: The correct answer is A. Selective IgA deficiency (<5 mg/dL) is the most common of all the primary immunodeficiency diseases. The incidence reported in the US has ranged from 1:250 to 1:1000. IgA has two subclasses, IgA1 and IgA2. IgA1 predominates in the serum, while IgA2 predominates in mucosal secretions as a dimer bound together by a J chain with a secretory piece attached. Recurrent bacterial and viral infections of the respiratory tract can be attributed to a lack of secretory IgA (sIgA), the predominant immunoglobulin of the mucosal immune system. IgD (choice B) has not been given any particular function other than to act as a receptor on the B cell. It can be found in very low levels in serum. IgG (choice C) is the major immunoglobulin found in the humoral immune response. A patient with a low IgG will experience pyogenic infections. IgM (choice D) is found in the early response to an antigen. If the patient was deficient in IgM he would have also been characteristically low in IgG and would have experienced recurrent pyogenic infection, usually commencing by the age of 5-6 months. Which of the following class II antigens would be most likely to play a contributing role in hay fever? A. DR2 B. DR3 C. DR4 D. DR5 E. DR7 Explanation: The correct answer is A. Hay fever is closely associated with DR2 (relative risk 19). DR2 is also associated with narcolepsy and multiple sclerosis. DR3 (choice B) is associated with Goodpasture's syndrome, celiac sprue, type 1 diabetes, and systemic lupus erythematosus. DR4 (choice C) is associated with pemphigus vulgaris, rheumatoid arthritis, and type 1 diabetes. DR5 (choice D) is associated with pernicious anemia and juvenile rheumatoid arthritis. DR7 (choice E) is associated with steroid responsive nephrotic syndrome. ▶ Reply to This usmle99 Permalink Reply by usmle99 on March 12, 2009 at 9:20pm A 46-year-old woman presents with complaints of feeling as if she has "sand in her eyes" and reports difficulty swallowing such foods as crackers or toast. Which of the following pairs of tests would likely yield positive results in this patient? A. Anti-centromere antibody and rheumatoid factor B. Anti-Scl-70 antibody and anti-Smith antibody C. Anti-Smith antibody and anti-double stranded DNA antibody D. Rheumatoid factor and anti-double stranded DNA E. Rheumatoid factor and anti-SS-A antibody Explanation: The correct answer is E. This patient has Sjögren's syndrome, which is an autoimmune disease characterized by lymphocytic infiltration of exocrine glands resulting in dry mouth (xerostomia) and dry eyes (keratoconjunctivitis sicca). Patients have an increased risk of malignant lymphoma. Autoantibodies produced include anti-Ro (SS-A), anti-La (SS-B), antinuclear antibodies, and rheumatoid factor. Choice A: Anti-centromere antibodies are a very specific marker for CREST syndrome. Rheumatoid factor is usually positive in rheumatoid arthritis but may be seen in low titers in patients with other autoimmune diseases and chronic inflammatory conditions. Choice B: Anti-Scl-70 antibodies are seen in patients with scleroderma. Anti-Smith antibody is seen in systemic lupus erythematosus. Choice C: Anti-Smith antibody and anti-double stranded DNA antibody are associated with systemic lupus erythematosus. Choice D: Rheumatoid factor is seen in approximately 80% of patients with rheumatoid arthritis and in low titers in other autoimmune disorders. Anti-double stranded DNA (anti-dsDNA) is seen in patients with SLE. A 12-year-old girl presents with a skin abscess. The causative organism is found to be Staphylococcus aureus. Over the past year, she has had several similar abscesses, as well as two bouts of aspergillosis. Which of the following is the most likely explanation for her repeated infections? A. Defective chemotactic response B. Defective NADPH oxidase C. Deficiency of C5 D. IgA deficiency E. Thymic hypoplasia Explanation: The correct answer is B. A history of recurrent infection with S. aureus and S. aspergillus suggests a diagnosis of chronic granulomatous disease, which is caused by a defect in the NADPH oxidase of neutrophils, resulting in an inability to generate toxic oxygen metabolites following phagocytosis. Patients become susceptible to certain opportunistic infections, particularly those caused by S. aureus, S. aspergillus, Nocardia, and Salmonella. Defective chemotactic response (choice A), seen in diseases such as Chédiak-Higashi syndrome and lazy-leukocyte syndrome, would make patients more susceptible to infections by bacteria such as Staphylococci and Streptococci. Deficiency of C5 (choice C), a complement component, would make patients prone to infection by gram-negative bacteria, especially Neisseria species. IgA deficiency (choice D) represents the most common immunodeficiency and would present with increased predisposition to autoimmune disorders, respiratory infections, and milk allergy. Thymic hypoplasia (choice E) is caused by defective development of the 3rd and 4th pharyngeal pouches. The result is decreased T-cell levels, making patients more prone to viral and fungal infections. They also will exhibit hypocalcemia and other signs of parathyroid insufficiency. A 4-month-old male presents with twitching of the facial muscles. He has previously been seen for several severe episodes of Candida infections. On examination, the child has low-set ears, hypertelorism, and a shortened philtrum. What additional findings would be likely in this individual? A. Absent thymic shadow on chest x-ray B. Decreased alpha-fetoprotein C. Decreased IgA levels D. Elevated IgM levels E. Prominent telangiectasias around the eyes Explanation: The correct answer is A. The clinical findings describe DiGeorge syndrome. Patients clinically present with tetany (usually first noted in the facial muscles) due to hypocalcemia secondary to hypoparathyroidism. The thymus is absent, as are the parathyroid glands, due to failure of development of the 3rd and 4th pharyngeal pouches. Recurrent infections due to defective cellular immunity and abnormal facies are additional features. Decreased alpha-fetoprotein (choice B) is an amniotic fluid marker for Down's syndrome. Down's syndrome patients have abnormal immune responses that predispose them to serious infections (particularly of the lungs) and to thyroid autoimmune disease. However, there is no defect of the parathyroid glands. Decreased IgA levels (choice C) describes selective IgA deficiency, which is the most common hereditary immunodeficiency. The syndrome is due to a failure of heavy-chain gene switching in B cells. Elevated IgM (choice D) is seen in hyper-IgM syndrome. Patients have a high concentration of IgM and normal numbers of T and B cells, but low levels of IgG, IgA, and IgE. Helper T cells have a defect in the surface protein CD40 ligand that interacts with CD40 on the B-cell surface. This results in an inability of the B cell to switch from the production of IgM to other classes of antibodies. Prominent telangiectasias around the eyes (choice E) are seen as part of the ataxia-telangiectasia syndrome. This is an autosomal recessive disorder, and is also referred to as a chromosomal breakage syndrome. Ataxia-telangiectasia is associated with increased numbers of translocations, especially involving the T-cell receptor loci; the gene for this disorder has been mapped to chromosome 11. Patients have an increased incidence of malignancy. Anti-ribonucleoprotein (anti-RNP) high titer Rheumatoid factor (RF) low titer Anti-single stranded DNA (anti-ssDNA) low titer Anti-double-stranded DNA (anti-dsDNA) not detected Anti-Smith antigen (anti-Sm) not detected Anti-SCI-70 not detected Which of the listed diseases is suggested by the antibody studies above? A. CREST syndrome B. Diffuse scleroderma C. Drug-induced lupus D. Mixed connective tissue disease E. Systemic lupus erythematosus Explanation: The correct answer is D. Mixed connective tissue disease (MCTD) is clinically an overlap autoimmune disorder, including joint pain, myalgias, pleurisy, esophageal dysmotility, and skin disease. Interestingly, the condition is usually characterized by high antibody titers to ribonucleoprotein (RNP), which serves as a disease marker, and the condition is apparently immunologically distinct from other connective tissue disorders. Low titer RF and anti-ssDNA are common features of MCTD, but other autoantibodies are less commonly observed. Anti-centromere antibody is the distinctive marker for the CREST (choice A) variant of scleroderma, characterized by calcinosis, Raynaud's phenomenon, esophageal dysmotility, sclerodactyly, and telangiectasia. Anti-SCI-70 is the distinctive marker for diffuse scleroderma (choice B), or systemic sclerosis, a multisystem disorder in which fibrosis of the skin, blood vessels, and viscera occurs. High titers of anti-histone antibody without other autoantibodies is the distinctive marker for drug-induced lupus (choice C). This disorder is particularly associated with administration of hydralazine and procainamide. High titers of anti-dsDNA is the distinctive marker for systemic lupus erythematosus (choice E). A 32-year-old female who is 6 months pregnant presents for prenatal care. A routine evaluation is performed, including testing for HIV antibody. The patient is reported to be negative for RPR, but positive for HIV antibody by the enzyme-linked immunoassay (EIA). The HIV Western blot is positive for antibody to the p24 antigen. The patient should be counseled A. that she and her baby were both infected with HIV B. that she had a false-positive HIV EIA C. that she is negative for HIV since the RPR was negative D. that she is positive for the HIV virus E. to have an HIV polymerase chain reaction (PCR) test performed Explanation: The correct answer is E. A patient who is HIV EIA-positive must always have the result confirmed by a confirmatory assay (e.g., HIV Western blot). The HIV Western blot is considered positive when the patient demonstrates the presence of antibody to at least two of three important HIV antigens, which are gp120, gp41, and p24. If no reaction is observed, then the patient is considered negative, but any reaction that is not consistent with a positive is reported as indeterminate. Therefore, this patient is considered indeterminate. The physician can wait 6 months and retest by Western blot; if the results are identical, then the patient is reported as negative, or the patient can be tested by another confirmatory test such as the PCR assay. A negative PCR in this situation would classify this patient as negative; however, it would be wise to retest the patient in 3-6 months if she had risk factors. Approximately 30% of the babies from untreated and 8% from treated HIV-positive mothers will be infected. One cannot conclude from the available data that both she and her baby are infected (choice A). It is possible that this patient had a false-positive HIV EIA assay, but with the present data, it is impossible to know if the patient is in the early stages of seroconversion or if the test result is a false positive (choice B). RPR (choice C) is a test for syphilis (and not a very specific one at that), not a test for HIV. Because this patient's Western blot was indeterminate, a confirmatory test (e.g., Western blot or PCR) must be performed to determine whether she is truly positive for the HIV virus (choice D). A 29-year-old Mexican American woman receives an intradermal tuberculin injection and later develops an indurated, erythematous papule 12 mm in diameter. This reaction is an example of which of the following? A. Antibody-dependent cell-mediated cytotoxicity B. Local anaphylaxis C. T-cell mediated cytotoxicity D. Type III hypersensitivity E. Type IV hypersensitivity Explanation: The correct answer is E. The tuberculin reaction is an example of delayed-type hypersensitivity (a form of Type IV hypersensitivity) in which the bulk of the tissue damage is done by macrophages that are stimulated by a few previously sensitized CD4+ memory T-cells recognizing antigens presented by the macrophages. In contrast, in T-cell mediated cytotoxicity (choice C, another form of Type IV sensitivity) the damage is done by CD8+ cytotoxic T-cells that recognize "foreign" cell surface antigens and directly lyse targeted cells. Antibody-dependent cell-mediated cytotoxicity reactions (choice A, a form of Type II hypersensitivity) involves cells coated with a thin layer of antibody that triggers attack by cells (monocytes, neutrophils, eosinophils, and natural killer cells) that can bind to Fc receptors. Local anaphylaxis (choice B, a form of Type I hypersensitivity) is due to the release of vasoactive substances by mast cells and basophils stimulated by memory (CD4+) T-cells reacting to antigen. Type III (choice D) hypersensitivity is due to deposition of circulating antigen-antibody complexes, often in small blood vessels. Which of the following could prevent an allergen from reacting with a specific IgE molecule present on the mast cell membrane? A. Antihistamine B. Blocking antibody C. Cromolyn sodium D. Epinephrine E. Theophylline Explanation: The correct answer is B. Blocking antibody is generally an IgG antibody against the allergen. It is induced in the allergic patient by administering small amounts of allergen over a period of time. When the person is again exposed to the allergen, the IgG reacts with the allergen before it can reach the IgE-coated mast cell. Antihistamines (choice A) would block histamine receptors but would not react with the allergen before it could reach the IgE-coated mast cell. Cromolyn sodium (choice C) is a drug that stabilizes mast cell membranes, thus inhibiting degranulation, but would have no effect on allergen binding to IgE on mast cells. Epinephrine (choice D) is the mainstay of therapy in severe cases of immediate hypersensitivity (anaphylaxis). It increases intracellular cyclic AMP, thus decreasing mast cell degranulation, and causes smooth muscle dilation in the airways. Epinephrine would have no effect on the binding of the allergen to IgE on mast cells. Cyclic AMP is degraded in cells by the enzyme phosphodiesterase. Theophylline (choice E) inhibits this enzyme, thereby increasing mast cell cyclic AMP levels and making degranulation less likely without affecting the binding of the allergen to IgE on mast cells. 1. Degranulation of mast cells and basophils occurs when allergen binds to two adjacent IgE antibodies on the membrane of the cell 2. Interleukin 4 released by CD4 helper T cells causes activated B cells to switch from making IgM to IgE 3. Processed allergen is associated with a class II molecule on an antigen presenting cell and recognized by CD4+ TH2 cells 4. Prostaglandins and leukotrienes are released 5. The allergen is phagocytized and processed by a macrophage or a dendritic cell Based on the numbered list above, which of the following is the correct sequence of events in the Type I hypersensitivity reaction? A. 1,2,3,5,4 B. 1,4,3,5,2 C. 1,5,2,3,4 D. 5,3,1,4,2 E. 5,3,2,1,4 Explanation: The correct answer is E. This question illustrates an important point. An allergen or antigen must first be processed by an antigen-presenting cell in an unsensitized host before the CD 4+ T helper cells can react to it. These cells of the TH2 subclass then produce a variety of cytokines including interleukin 4. Interleukin 4 causes the heavy chain switching in the B cells from IgM to IgE. The IgE binds to the surface of mast cells or basophils by the Fc fragment. When the patient is re-exposed to the sensitizing allergen, the IgE binds to the allergen and causes degranulation of the mast cell or basophil. Which of the following cell surface markers is required for lysis of IgG-coated target cells (antibody-dependent, cell-mediated cytotoxicity, or ADCC) by natural killer cells? A. CD3 B. CD16 C. CD19 D. CD21 E. CD56 Explanation: The correct answer is B. CD16 is a cell surface marker used to identify natural killer (NK) cells (lymphocytes lacking most T- and B-cell markers). CD16 is an Fc receptor for IgG, allowing the NK cells to bind to the coated target cell during ADCC, facilitating lysis. CD3 (choice A) is a five-polypeptide cluster that represents the non-variable part of the T-cell receptor complex. The variable part is able to rearrange itself to adapt to specific antigens. Natural killer cells are CD3-negative. CD19 (choice C) is a B-cell marker. It is a signal-transducing molecule that is expressed in early B-cell differentiation. NK cells are negative for CD19. C21 (choice D) is also a B-cell marker. It is a complement receptor, and is also the same receptor the Epstein-Barr virus uses to bind to cells during infection. CD56 (choice E) is a natural killer cell marker, but is not involved with ADCC. A 15-month-old boy with recurrent episodes of ear, skin, and respiratory tract infections is found to have extremely low levels of serum IgG. Which of the following findings would support a diagnosis of common variable immunodeficiency in this child? A. Absent germinal centers in lymph nodes B. Depressed levels of IgA and IgM C. Depressed numbers of circulating B lymphocytes D. Hypocalcemia E. Reversal of CD4:CD8 ratio Explanation: The correct answer is B. Common variable immunodeficiency is a disease caused by abnormal B-cell differentiation, characterized by decreased numbers of plasma cells and inadequate immunoglobulin production. The majority of patients with common variable immunodeficiency have low levels of all antibody classes. Lymph node architecture is unaltered in common variable immunodeficiency, although plasma cells are not present in the medullary cords. In contrast, normal germinal centers are absent (choice A) in X-linked agammaglobulinemia and some cases of severe combined immunodeficiency disease. Patients with common variable immunodeficiency have normal numbers of circulating B lymphocytes. B-cell deficiency (choice C) is more typical of X-linked agammaglobulinemia. Hypocalcemia (choice D) and immunodeficiency are features seen in DiGeorge syndrome, in which the thymus and parathyroids are absent due to failure of development of the pharyngeal pouches. The parathyroids and thymus are normal in common variable immunodeficiency. The CD4:CD8 ratio is reversed (choice E) in HIV disease, due to selective destruction of CD4+ T cells. T cells are not affected by common variable immunodeficiency. A 24-year-old man presents with complaints of itching on his arms and face. Physical examination reveals well-circumscribed wheals with raised, erythematous borders and blanched centers. Which form of hypersensitivity is this patient probably exhibiting? A. Acute serum sickness (Type III) B. Antibody-dependent cell-mediated cytotoxicity (Type II) C. Anti-receptor antibodies (Type II) D. Delayed type hypersensitivity (Type IV) E. Immediate type hypersensitivity (Type I) Explanation: The correct answer is E. Urticaria (hives) is a good example of a local anaphylaxis reaction, which is classified as a Type I hypersensitivity reaction. Type I hypersensitivity reactions involve preformed Ig E antibody bound to mast cells or basophils, which release vasoactive and spasmogenic substances when they react with antigens. Acute serum sickness (choice A) is now uncommon but was formerly seen when animal sera were used for passive immunization. The eosinophil-mediated cytotoxicity against parasites is an example of antibody-dependent cell-mediated cytotoxicity (choice B). Myasthenia gravis is an example of a disease caused by anti-receptor antibodies (choice C). The tuberculin (PPD) reaction used to test for tuberculosis exposure is an example of delayed-type hypersensitivity (choice D). A 47-year-old woman presents to a physician with finger stiffness. Physical examination demonstrates marked thickening of the skin, most striking on the hand, which is limiting finger mobility. The physical examination also reveals multiple small telangiectasias and several hard nodules on the buttocks. Questioning of the patient reveals recent difficulty swallowing and skin color changes when the hands are exposed to cold. Autoantibody formation to which of the following substances is most strongly associated with this patient's condition? A. Centromeres B. Double-stranded DNA C. Glomerular basement membrane D. Scl 70 E. Smith antigen Explanation: The correct answer is A. The patient has the CREST variant of scleroderma, which is associated with anti-centromere antibody. The CREST syndrome comprises calcinosis (the hard calcified, subcutaneous nodules of the buttocks), Raynaud's phenomenon (the skin color changes in response to cold), esophageal dysmotility (difficulty swallowing), sclerodactyly (scleroderma involving the fingers), and telangiectasia (small vascular lesions of the skin). The CREST variant usually has a more benign course than the systemic variant of scleroderma. Anti-double-stranded DNA (choice B) and anti-Smith antigen (choice E) are associated most strongly with systemic lupus erythematosus. Anti-glomerular basement membrane (choice C) is associated with Goodpasture's disease. Anti-Scl 70 (choice D) is associated with the systemic form of scleroderma. A 54-year-old man presents with complaints of shortness of breath, a sore tongue, and a pins-and-needles sensation in his feet. Laboratory examination reveals macrocytosis, anemia, and hypersegmented neutrophils. Antibodies to intrinsic factor are detected in the patient's serum. Which of the following class II antigens would be most likely to play a contributing role in the etiology of this patient's disease? A. DR2 B. DR3 C. DR4 D. DR5 E. DR7 Explanation: The correct answer is D. The disease with autoantibodies to intrinsic factor is pernicious anemia (atrophic gastritis and megaloblastic anemia secondary to vitamin B12 deficiency). Pernicious anemia is associated with the DR5 class II antigen (relative risk 5); DR5 is also associated with juvenile rheumatoid arthritis. DR2 (choice A) is associated with allergy, multiple sclerosis, and narcolepsy. DR3 (choice B) is associated with Goodpasture's syndrome, celiac sprue, Type 1 diabetes mellitus, and systemic lupus erythematosus. DR4 (choice C) is associated with pemphigus vulgaris, rheumatoid arthritis, and Type 1 diabetes mellitus. DR7 (choice E) is associated with steroid-responsive nephrotic syndrome. A 35-year-old male presents to an infectious disease specialist with recurrent infections with encapsulated bacterial organisms. The history indicates that these infections have become apparent in the last 6 months. Laboratory findings indicates that the total immunoglobulin level exceeded 900 mg/dL with a low CH50 (hemolytic complement) level. Which of the following is the correct diagnosis? A. Acquired hypogammaglobulinemia (common variable hypogammaglobulinemia) B. C3 deficiency C. Hyper-IgM syndrome D. Wiskott-Aldrich syndrome E. X-linked infantile hypogammaglobulinemia Explanation: The correct answer is B. Individuals with C3 deficiency have an increased susceptibility to recurrent bacterial infections, especially with encapsulated bacteria. This susceptibility illustrates the important role of C3 as an opsonin. C3 deficiency is often not detected until later life. Acquired hypogammaglobulinemia, also known as common variable hypogammaglobulinemia, (choice A) is ruled out by the fact that the total immunoglobulin level is greater than 300 mg/dL. In hyper-IgM syndrome (choice C) there is very little, if any, isotype switching, resulting in patients who are IgG and IgA deficient, but synthesize large amounts of polyclonal IgM. These patients are susceptible to pyogenic infections, and thus the disorder is detected very early in life. In Wiskott-Aldrich syndrome (choice D), a combined immunodeficiency syndrome, immune abnormalities are apparent at birth. Patients usually have low IgM levels with elevated levels of serum IgA and IgE; recurrent pyogenic infections, eczema, and thrombocytopenia are characteristic Patients with X-linked infantile hypogammaglobulinemia (choice E) are detected in the first 4-8 months of life and have total immunoglobulin levels less than 200 mg/dL. Recurrent pyogenic infections are characteristic of this disorder. A 45-year-old homeless man has a chronic cough, a cavitary lesion of the lung, and is sputum positive for acid-fast bacilli. Which of the following is the principal form of defense by which the patient's body fights this infection? A. Antibody-mediated phagocytosis B. Cell-mediated immunity C. IgA-mediated hypersensitivity D. IgE-mediated hypersensitivity E. Neutrophil ingestion of bacteria Explanation: The correct answer is B. The principal host defense in mycobacterial infections (such as this patient's tuberculosis) is cell-mediated immunity, which causes formation of granulomas. Unfortunately, in tuberculosis and in many other infectious diseases characterized by granuloma formation, the organisms may persist intracellularly for years in the granulomas, only to be a source of activation of the infection up to decades later. While antibody-mediated phagocytosis (choice A) is a major host defense against many bacteria, it is not the principal defense against Mycobacteria. IgA-mediated hypersensitivity (choice C) is not involved in the body's defense against Mycobacteria. IgE-mediated hypersensitivity (choice D) is not involved in the body's defense against Mycobacteria. It is important in allergic reactions. Neutrophil ingestion of bacteria (choice E) is a major host defense against bacteria, but is not the principal defense against Mycobacteria. Which marker or markers is present on B cells and could be used to specifically identify such cells in a flow cytometric analyzer? A. CD 3 B. CD 8 C. CD 14 D. CD 16 and CD 56 E. CD 19 and CD 20 Explanation: The correct answer is E. B cells have IgM and IgD on their surfaces, which are excellent markers for B cells. In addition, B cells have CD 19, 20, and 21 markers that aid in their identification. The CD 3 (choice A) marker is present on all T cells. The CD 8 (choice B) marker is present on cytotoxic T lymphocytes. The CD 14 (choice C) marker is used to identify macrophages. The CD 16 and CD 56 (choice D) markers are present on natural killer (NK) cells. A 26-year-old systems analyst presents for evaluation of a bee sting allergy. He describes an episode in which he was stung on the forearm by a bee and, within 5 minutes, experienced pruritus, urticaria, and mild wheezing. The effector cell in this type of hypersensitivity is a(n) A. eosinophil B. mast cell C. megakaryocyte D. neutrophil E. TH1 CD4+ lymphocyte Explanation: The correct answer is B. This patient is experiencing the early phase of type I hypersensitivity, characterized by pruritus and watery discharge from the nose, bronchospasm, mucus secretion in the airways, and a wheal-and-flare response with pruritus in the skin. The mechanism of hypersensitivity involves prior sensitization of a population of TH2 cells that produced cytokines, including interleukin-4. The interleukin-4 causes B cells to switch their heavy chain class from IgM to IgE. The IgE molecules then attach to mast cells or basophils. With subsequent antigen (allergen) challenge, the mast cells degranulate and release mediators, including histamine, which produces the anaphylactic response. Eosinophils (choice A) are involved in type I hypersensitivity reactions as well as type II antibody-dependent cell cytotoxicity reactions directed against parasites. Eosinophils enter the area of the reaction due to the release of eosinophil chemotactic factor (eotaxin) and the beta-chemokine RANTES released from TH2 CD4+ cells and mast cells. Eosinophils are recruited into the tissues as part of the late-phase reaction of type I hypersensitivity. Their survival in tissue is dependent upon IL-3, IL-5, and granulocyte-monocyte colony stimulating factor (GM-CSF) released from TH2 cells. Megakaryocytes (choice C) are bone marrow cells that produces platelets. They are not involved in type I hypersensitivity. Neutrophils (choice D) are not a cell type that is key to the development of type I hypersensitivity. TH1 CD 4+ lymphocytes (choice E) are associated with delayed type hypersensitivity reactions such as contact dermatitis involving exposure to poison ivy or poison oak. A 3-year-old male who is small for his age presents with a history of pyogenic infections. Physical examination is remarkable for a high fever, hepatosplenomegaly, and inguinal and cervical lymphadenopathy. A culture of a purulent discharge from an abscess grows out Staphylococcus aureus. Immunoglobulin and complement levels are normal. The boy received all of the standard immunizations without any adverse effects. The boy's immune deficiency most likely involves which of the following? A. B cells B. Chemotaxis C. IgG subclass 2 D. Phagocytic cells E. T cells Explanation: The correct answer is D. This patient has chronic granulomatous disease (CGD), which is associated with a defective intracellular respiratory burst in phagocytes. CGD consists of a group of heterogenous disorders of oxidative metabolism affecting the pathways required for hydrogen peroxide production by phagocytic cells. It is inherited as an X-linked or autosomal recessive trait. Normally, stimulated phagocytic cells undergo a respiratory burst consisting of increased oxygen consumption leading to the generation of intracellular hydrogen peroxide and superoxide radicals. The hydrogen peroxide and superoxide radicals are required for the killing of ingested intracellular organisms. The reaction is catalyzed by an NADPH oxidase that appears to be defective in the phagocytes of patients with CGD; the enzyme defect appears to be due to inherited mutations in the genes encoding cytochrome subunits. In CGD patients, the engulfment process by the phagocytic cells is normal; however, the pathogenic organism will not be killed, but will persist within the cell. The patients suffer from infections with organisms that are normally considered of low virulence (e.g., Staphylococcus aureus, Aspergillus, Candida, Escherichia coli, and Serratia marcescens). The nitroblue tetrazolium (NBT) test is used to screen for CGD. B cells (choice A) are responsible for antibody-mediated immunity, and the immunoglobulin levels in this patient were normal. Chemotaxis (choice B) is important in the migration of the phagocytic cell toward the site of infection, not in intracellular killing. IgG subclass 2 (choice C) is the most important immunoglobulin in the protection against encapsulated organisms. T cells (choice E) are important in the host response to viruses, fungi, and intracellular bacterial organisms. The patient was immunized normally (including the live, attenuated MMR vaccine), making T-cell dysfunction unlikely. A 20-year-old healthy married Caucasian male undergoes a splenectomy following a severe motorcycle accident. He is started on a prophylactic course of antibiotics following surgery, which he discontinues 3 months later. 9 months after the splenectomy, he presents to the emergency room with shortness of breath, pleuritic chest pain, chills and fever, and is diagnosed with lobar pneumonia. Which of the following is the most likely organism? A. Chlamydia pneumoniae B. Pneumocystis carinii C. Pseudomonas aeruginosa D. Staphylococcus aureus E. Streptococcus pneumoniae Explanation: The correct answer is E. The spleen is very effective in the removal of encapsulated organisms such as Streptococcus pneumoniae, Haemophilus influenzae and Neisseria meningitidis. The capsular polysaccharides of S. pneumoniae are highly antigenic and type-specific. Type-specific anticapsular antibodies to these T-independent antigens result in effective opsonization and host recovery. In untreated S. pneumoniae infections, recovery clearly is due to opsonizing antibody. Splenectomized patients may have low levels of IgG2 and IgM because splenic lymphocytes are the major producers of these antibodies, which provide protection against encapsulated organisms. Splenectomy therefore renders patients extremely susceptible to infections with encapsulated organisms; immunization against S. pneumoniae is therefore recommended. Chlamydia pneumoniae(choice A) causes a community-acquired pneumonia. However, the patient has no reason for increased susceptibility to this organism (the organism is not encapsulated). Pneumocystis carinii (choice B) is common in patients with acquired immune deficiency. A clinical and social history would quickly indicate the likelihood of such an infection. Pneumocystis is currently thought to be a fungus, although previously it was believed to be a protozoan. Pseudomonas aeruginosa(choice C) is typically a cause of hospital-acquired pneumonia (and is not encapsulated). Staphylococcus aureus (choice D) is typically a cause of hospital-acquired pneumonia (and is not encapsulated). A Mexican immigrant presents with a 3-month history of weight loss, night sweats, a productive cough with blood-tinged sputum, anorexia, general malaise, and a low grade fever. A PPD skin test shows > 10 mm of induration. If the area of induration were biopsied, which of the following type of reactive cells would be found? A. B lymphocyte B. Eosinophil C. Mast cell D. Neutrophil E. T lymphocyte Explanation: The correct answer is E. The CD4+ population of T lymphocytes, specifically TH1 cells, are responsible for the delayed hypersensitivity reaction seen with a skin test in a previously sensitized patient. The clinical pattern in the test question is classic for reactivation or adult-type tuberculosis. One of the risk factors to consider in evaluating a patient for tuberculosis is a foreign-born patient who has recently immigrated to the U.S. A 10 mm reaction on a skin test is considered positive if the person is in a high-incidence group (foreign born, medically underserved, low-income population, or resident of a long-term care facility). B lymphocytes (choice A) are involved in humoral immune reactions. Antibody production is not a feature of tuberculosis hypersensitivity. Eosinophils (choice B) are important in type I hypersensitivity reactions and in immune mediated responses to parasitic infections (ADCC). They are not associated with tuberculosis hypersensitivity. Mast cells (choice C) are tissue cells which are involved in type I hypersensitivity reactions. They have surface receptors for the Fc fragment of the IgE molecule. Neutrophils(choice D) are associated with acute inflammatory reactions. They are considered a "non-specific" cell in that they do not interact with an antigen. The neutrophil is not a classic cell type seen in tuberculosis hypersensitivity. A child has a history of recurrent infections with organisms having polysaccharide antigens (i.e., Streptococcus pneumoniae and Hemophilus influenzae). This susceptibility can be explained by a deficiency of A. C3 nephritic factor B. C5 C. IgG subclass 2 D. myeloperoxidase in phagocytic cells E. secretory IgA Explanation: The correct answer is C. IgG is the predominant antibody in the secondary immune response. IgG subclass 2 is directed against polysaccharide antigens and is involved in the host defense against encapsulated bacteria. C3 nephritic factor (choice A) is an IgG autoantibody that binds to C3 convertase, making it resistant to inactivation. This leads to persistently low serum complement levels and is associated with Type II membranoproliferative glomerulonephritis. C5 (choice B) is a component of the complement system. C5a is an anaphylatoxin that effects vasodilatation in acute inflammation. It is also chemotactic for neutrophils and monocytes and increases the expression of adhesion molecules. A deficiency of C5a would affect the acute inflammatory response against any microorganism or foreign substance. Myeloperoxidase in phagocytic cells (choice D) is an element of the oxygen-dependent pathway present in phagocytic cells that effectively kills bacterial cells. The hydrogen peroxide-halide complex is considered the most efficient bactericidal system in neutrophils. Chronic granulomatous disease is associated with a deficiency of NADPH oxidase, which converts molecular oxygen to superoxide (the first step in the myeloperoxidase system). Patients are susceptible to granulomatous infections and staphylococcal infections. Secretory IgA (choice E) is the immunoglobulin associated with mucous membranes. Selective IgA deficiency is the most common hereditary immunodeficiency. In this disorder, there is failure of the B cell to switch the heavy chain class from IgM to IgA. Patients have an increased incidence of sinopulmonary infections, diarrhea, allergies, and autoimmune diseases. Which of the following is the primary opsonin in the complement system? A. C1q B. C3b C. C5 D. C5a E. Factor B Explanation: The correct answer is B. C3b is the most critical molecule in both the classical and alternative complement pathways. C3, the most abundant protein of all the complement proteins, is cleaved into C3a and C3b. C3b attaches to bacterial surfaces for opsonization by phagocytes. C3a binds to mast cells and basophils, activating them and producing histamine release. C1q (choice A) is a complement component in the classical pathway. It binds to the constant heavy domain of an IgG molecule that has reacted with the bacterial surface epitope. C1q is not involved in opsonization as it simply functions as an enzyme in the early complement cascade. C5 (choice C) is a protein, that once split into C5b, initiates the assembly of the membrane attack complex. This complex consists of C5b, C6, C7, C8, and polymerization of C9, and is responsible for lysis of the bacteria. C5a is a strong chemotactic molecule. This fragment is the result of C5 being split by the C5 convertases of both pathways. This C5a (choice D) is a strong chemotactic factor for neutrophils and results in stimulating the inflammatory response. Factor B (choice E) is an activator protein of the alternative pathway. It combines with C3b to form C3bBb. This C3bBb is the C3 convertase of the alternative pathway. ▶ Reply to This usmle99 Permalink Reply by usmle99 on March 12, 2009 at 9:21pm A 27-year-old woman presents to the emergency department complaining of wheezing and swelling of the lips. The symptoms began acutely just after eating a chicken and nut dish at a nearby Chinese restaurant. She also complains of tightness in her chest and difficulty breathing. On examination, she has a temperature of 36.8 C, a heart rate of 110/min, a blood pressure of 90/50 mm Hg, and a respiratory rate of 24/min. She is anxious and in mild respiratory distress. There is diffuse facial erythema, with swelling of the lips and tongue. Wheezing is noted bilaterally, and oxygen saturation is 92%. Which of the following cytokines is necessary for production of the antibody that mediates this response? A. Interferon-γ B. Interleukin 1 C. Interleukin 2 D. Interleukin 3 E. Interleukin 4 F. Interleukin 5 G. Interleukin 6 Explanation: The correct answer is E. This is a case of immediate (type I) hypersensitivity mediated by IgE, mast cells, and basophils. The cytokine that mediates the isotype switch to IgE in B lymphocytes is interleukin 4, produced by T-helper-2 cells. Interferon-γ(choice A) is a cytokine that activates macrophages and would not be involved in this response. Interleukin 1 (choice B) is an endogenous pyrogen involved in the production of the acute phase reaction of systemic inflammation. Interleukin 2 (choice C) is the cytokine that induces T cell proliferation. Interleukin 3 (choice D) is a hematopoietic colony stimulating factor. Interleukin 5 (choice F) is an eosinophil stimulating factor. Interleukin 6 (choice G) is another endogenous pyrogen and a stimulator of B cells, but not an inducer of isotype switch to IgE production. [IMG]file:///C:/DOCUME%7E1/Owner/LOCALS%7E1/Temp/msohtml1/01/clip_image002.gif[/IMG] . Which of the following characteristics most strongly suggests that the cell depicted above is a phagocytically active macrophage? A. Immunocytochemical detection of collagenase B. Microvilli-covered surface C. Presence of receptors for IgG and complement D. Presence of secondary lysosomes throughout cytoplasm E. Shape of the nucleus Explanation: The correct answer is C. Macrophages interact with a segment of the heavy chains of the IgG molecule, the Fc domain. Thus, the macrophage receptors for IgG are called Fc receptors. Complement receptors are also found on the macrophage surface. Phagocytosis occurs more readily when both of these components are present. Neutrophils and eosinophils have similar receptors: neutrophils assist macrophages in destruction of bacteria, while eosinophils respond to parasitic infections. Collagenase can be demonstrated immunologically (choice A) in a number of cells including eosinophils, mast cells, osteoclasts, and fibroblasts. Microvilli (choice B) or pseudopodial extensions are common in macrophages, but also may be found on other cells such as fibroblasts and epithelial cells that phagocytize material. Secondary lysosomes (choice D) are a feature of macrophages, but may also be found in fibroblasts and epithelial cells. The shape of the nucleus (choice E) has little to do with phagocytic activity. A 3-year-old male presents with a skin rash and epistaxis. He has had several, severe sinopulmonary infections. A careful history reveals that his maternal uncle died of bleeding complications following an emergency cholecystectomy. What additional findings are likely in this case? A. A CD4/CD8 ratio of < 1.5:1 B. Cerebellar ataxia C. Elevated platelet count and high serum IgG, IgA, and IgE levels D. Low platelet count and low serum IgG levels E. Low platelet count and low serum IgM levels Explanation: The correct answer is E. This clinical scenario is typical for Wiskott-Aldrich syndrome. This is a sex-linked recessive disorder presenting with the triad of thrombocytopenia, eczema, and recurrent sinopulmonary infections. Serum IgM levels are low but IgG, IgA, and IgE levels are increased. Patients have a defective response to polysaccharide antigens, which is due to a cytoskeletal defect in T cells that inhibits their binding to B cells. A CD4/CD8 ratio of < 1.5:1 (choice A) is the pattern seen in AIDS due to selective tropism of the CD4+ T-helper cell population. Cerebellar ataxia (choice B) is part of the ataxia-telangiectasia syndrome. The ataxia develops between age 2 and 5. The defect is associated with a DNA repair enzyme deficiency. Elevated platelet count and increased serum levels of IgG, IgA, and IgE (choice C) is not associated with a particular syndrome. Polyclonal gammopathies result in an increase in immunoglobulin of more than one class. This benign alteration is frequently seen in viral or bacterial infections. Thrombocythemia may be associated with a myeloproliferative syndrome or with a secondary reactive process. Low platelet count and low serum IgG (choice D) are not associated with any particular disorder. A 4-year-old child presents to a physician with purpura. Questioning of the mother reveals that the child also has a history of eczema and recurrent pneumococcal pneumonia. Blood studies demonstrate thrombocytopenia and a selective decrease in IgM. Patients with this disorder have a 12% chance of developing which of the following potentially fatal malignancies? A. Astrocytoma B. Chronic myelogenous leukemia C. Colon carcinoma D. Non-Hodgkin's lymphoma E. Wilms' tumor Explanation: The correct answer is D. The patient has Wiskott-Aldrich syndrome, which is an X-linked partial combined immunodeficiency disorder. It presents clinically as the triad of thrombocytopenic purpura, eczema, and recurrent opportunistic infections with organisms having polysaccharide capsules, such as Streptococcus pneumoniae. The patients characteristically have decreased serum IgM and T-cell deficits (which appear later in life). Most patients die before the second decade of life, either from infection or from non-Hodgkin's lymphoma. Bone marrow transplantation has had some therapeutic success in these patients. None of the other neoplasms listed occurs with greater frequency in patients with Wiskott-Aldrich syndrome. A 20-year-old college student working part-time in a pediatric AIDS clinic develops a viral exanthem with a rash. Her blood is drawn and tested for specific antibodies to varicella-zoster (chickenpox). Anti-varicella immunoglobulin belonging to which of the following antibody classes would indicate that she is immune to chickenpox? A. IgA B. IgD C. IgE D. IgG E. IgM Explanation: The correct answer is D. The immunoglobulin that represents past exposure to an antigen is IgG. It is the predominant antibody in the secondary response and is an important defense against bacteria and viruses. In the secondary response, a much larger amount of IgG antibody is produced than IgM, and the levels of IgG tend to persist for a much longer time than in the primary response. IgA (choice A), the main immunoglobulin found in secretions, prevents the attachment of bacteria and viruses to mucosal surfaces. It does not bind complement. IgD (choice B) probably functions as an antigen receptor. It is found on the surface of many B lymphocytes and is found in small amounts in serum. IgE (choice C) is the primary antibody in type I hypersensitivity. It mediates the anaphylactic response and also participates in host defenses against parasites (antibody-dependent cell-mediated cytotoxicity-ADCC). IgM (choice E) is the chief immunoglobulin produced in the primary response to an antigen. It is also produced in the secondary response, but in lesser amounts than IgG. It exists as a pentamer, which affords a high degree of avidity of binding to antigens. It is produced by the fetus in certain infections, but does not cross the placenta. It does bind complement. A 12-year-old girl has a temperature of 102.5 F and a sore throat. Two days later, she develops a diffuse erythematous rash and is taken to her pediatrician. On physical examination, there is circumoral pallor, and an erythematous rash with areas of desquamation is noted. The myocardial damage that can follow this infection is produced in a manner similar to the damage associated with which of the following disorders? A. Atopic allergy B. Contact dermatitis C. Graft-vs.-host disease D. Graves disease E. Idiopathic thrombocytopenic purpura F. Myasthenia gravis G. Rheumatoid arthritis H. Serum sickness I. Systemic lupus erythematosus Explanation: The correct answer is E. This is a case of rheumatic fever, which is an immunologically mediated sequela to Streptococcus pyogenes pharyngitis. It is a type II cytotoxic hypersensitivity, involving antibodies that bind to cardiac tissue, activate complement, and thereby cause cell destruction. It is therefore most similar to idiopathic thrombocytopenic purpura, which is also a form of type II cytotoxic hypersensitivity, in this case mediated by antibodies against platelets producing complement fixation and causing the clotting dyscrasia. Atopic allergy (choice A) is a form of type I hypersensitivity, mediated by IgE antibodies and basophils and mast cells. Contact dermatitis (choice B) is a form of type IV hypersensitivity mediated by T cells and macrophages. Graft-vs.-host disease (choice C) is a form of type IV hypersensitivity mediated by T cells and macrophages. Graves disease (choice D) is a form of type II hypersensitivity, but it is NOT cytotoxic in its action. Instead, antibodies to the TSH receptors on thyroid cells cause overstimulation of the gland and its eventual exhaustion. Myasthenia gravis (choice F) is a form of type II hypersensitivity, but NOT of the cytotoxic variety. In this case, antibodies to the acetylcholine receptors on neurons diminish neurotransmission. Rheumatoid arthritis (choice G) is a form of type III hypersensitivity, caused by immune complex deposition in joints and subsequent activation of complement. Serum sickness (choice H) is a form of type III hypersensitivity, caused by immune complex deposition. Systemic lupus erythematosus (choice I) is a form of type III hypersensitivity, caused by immune complex deposition. A 32-year-old medical technician had a history of acute eczematous dermatitis on her hands and wrist in the distribution of the latex gloves she wore. The skin of her hands was dry, crusted, and thickened. The eczematous reaction cleared after a 2-week vacation. After 72 hours back on the job, the eczematous dermatitis returned and continued to grow worse. Which of the following characterizes the technician's reaction to the latex gloves? A. Irritant dermatitis B. Type I reaction C. Type II reaction D. Type III reaction E. Type IV reaction Explanation: The correct answer is E. Sensitization to latex has become a major healthcare problem. Local skin irritations are common but more severe allergic reactions occur, up to and including rare anaphylactic reactions that are occasionally fatal. The immune responses to latex are immediate-type hypersensitivity (type I) reactions, expressing themselves in minutes, and/or delayed-type hypersensitivity (type IV) reactions, which will express themselves in 48-72 hours. The type I reactions are due to the IgE-mediated sensitivity to latex proteins while the type IV reactions are due to a cell-mediated response to the chemicals that are added in the processing of latex. The type IV response in this circumstance would be referred to as contact dermatitis. Irritant dermatitis (choice A) can be observed in the early stages of sensitization and can be due to sweating, rubbing, and residual soap. The timing of the reaction after 72 hours points to the type IV reaction rather than an irritant dermatitis. A type I reaction (choice B) would have been apparent in minutes with characteristic rhinitis, conjunctivitis, urticaria, asthma, angioedema, or anaphylaxis after she put on her latex gloves. A type II reaction (choice C) is a cytolytic response mediated by an antibody. The type II reaction is observed in the context of hemolytic anemias, thrombocytopenia, neutropenia, etc. A type III reaction (choice D) is an immune-complex response that sets into motion an inflammatory response. PEDIGREE [IMG]file:///C:/DOCUME%7E1/Owner/LOCALS%7E1/Temp/msohtml1/01/clip_image004.jpg[/IMG] A couple brings their son in to a specialty clinic for evaluation of recurrent bacterial infections involving the respiratory tract. Other family members have a similar disorder, as noted in the pedigree above. Which of the following is the most likely diagnosis? A. Bruton's agammaglobulinemia B. Common variable immunodeficiency C. DiGeorge syndrome D. Hereditary angioedema E. Isolated IgA deficiency Explanation: The correct answer is A. Several immunodeficiency disorders have X-linked genetics, including Bruton's agammaglobulinemia, Wiskott-Aldrich syndrome, and some cases of severe combined immunodeficiency diseases. Bruton's agammaglobulinemia is characterized by recurrent respiratory infections caused by pyogenic organisms. Common variable immunodeficiency (choice B) is a relatively common, but probably heterogeneous, group of acquired and familial diseases. It is distinct from X-linked (Bruton's) agammaglobulinemia. DiGeorge syndrome (choice C) is due to a developmental malformation of the third and fourth pharyngeal pouches leading to failure of the thymus, and sometimes the parathyroids, to develop. Defective cellular immunity and abnormalities of calcium metabolism are typical. Hereditary angioedema (choice D) is a usually recessive genetic disease caused by deficiency of C1 esterase inhibitor. Isolated IgA deficiency (choice E) can be acquired or genetic, but is not usually X-linked. Which of the following cytokines is associated with the development of cell-mediated immunity? A. IL-3 B. IL-4 C. IL-5 D. IL-10 E. IL-I2 Explanation: The correct answer is E. Gamma interferon from T helper 1 CD4 + lymphocytes and IL-12 from macrophages are major stimulators of the development of a cell-mediated immune response to intracellular organisms such as tuberculosis, leishmaniasis, histoplasmosis, and many other organisms. IL-3 (choice A) is a cell growth factor produced by thymic epithelial cells and T cells to regulate different T cell differentiation processes. IL-4 (choice B) is produced by T helper 2 CD4 + lymphocytes and is involved in class switching to IgE. It also downregulates T helper 1 CD4 + lymphocytes. IL-5 (choice C) is produced by T helper 2 CD4 + lymphocytes and is involved in IgA production, eosinophil differentiation, and B cell differentiation. IL-10 (choice D) is produced by T helper 2 CD4 + lymphocytes. It downregulates the cell-mediated immune system. A 25-year-old female presents with pain and tenosynovitis of the wrists and ankles, and arthralgias of other joints. She notes two prior episodes similar to the present one. She just had her menstrual period during the previous week. Physical examination reveals ulcerated lesions overlying the wrists and ankles. These symptoms are likely due to deficiency of which of the following? A. C1 esterase inhibitor B. Ciliary function C. Complement (C6-C8) components D. Endothelial adhesion molecules E. Eosinophils Explanation: The correct answer is C. This patient has disseminated gonococcemia. Gonococcal arthritis and tenosynovitis typically involve both the upper and lower extremities equally. Vesicular skin lesions are characteristic of disseminated gonococcal disease. Females are at particular risk of gonococcemia during menstruation, since sloughing of the endometrium allows access to the blood supply, necrotic tissue enhances the growth of Neisseria gonorrhoeae, and there is an alteration of the pH. Patients who have a C6-C8 deficiency have an increased risk of disseminated gonococcemia and tend to have multiple episodes. These patients are also at risk for bacteremia from Neisseria meningitidis. C1 esterase inhibitor deficiency (choice A) can occur as an autosomal dominant disorder or is acquired. Patients have angioedema without urticaria. The syndrome is also associated with recurrent attacks of colic and episodes of laryngeal edema. Ciliary dysfunction (choice B) is a marker of Kartagener's syndrome (immotile cilia syndrome). The syndrome includes infertility, bronchiectasis, sinusitis, and situs inversus. It is an autosomal recessive disorder caused by abnormalities in the dynein arm of the cilia. Endothelial adhesion molecule deficiency (choice D), or beta2 integrin deficiency, is characterized by failure of neutrophils to express CD18 integrins on their surface. Patients have impaired phagocyte adherence, aggregation, chemotaxis, and phagocytosis of C3b-coated particles. Clinically, there is delayed separation of the umbilical cord, sustained agranulocytosis, recurrent infections of skin and mucosa, gingivitis, and periodontal disease. Eosinophil deficiency (choice E) or eosinopenia occurs with stressors such as acute bacterial infection and following administration of glucocorticoids. There is no known adverse effect of eosinopenia. A pregnant 38-year-old mother of two presents at the office concerned about her pregnancy. She is Rh-negative and her husband is Rh-positive. Both of her children are also Rh-negative. She has had two spontaneous abortions and carried a third pregnancy to term, but the child died at birth, diagnosed with erythroblastosis fetalis. The test of choice to determine the presence of circulating anti-Rh antibody in the mother is a(n) A. direct Coombs test to measure IgG anti-Rh antibody B. direct Coombs test to measure IgM anti-Rh antibody C. indirect Coombs test to measure IgG anti-Rh antibody D. indirect Coombs test to measure IgM anti-Rh antibody E. Ouchterlony test to measure precipitin IgG anti-Rh antibody Explanation: The correct answer is C. The indirect Coombs test would be the test of choice to detect the presence of IgG (warm agglutinin) anti-Rh antibody circulating in the mother's blood. Serum is collected from the mother and mixed with Rh-positive red blood cells, followed by incubation with complement. If the mother had circulating IgG anti-Rh, it would bind to the Rh antigen on the Rh-positive red blood cells, fixing the complement and lysing the target cells. A titer of greater than or equal to 1:8 is considered diagnostic. The direct Coombs test (choices A and B) would be used to determine if anti-IgG or anti-C3 antisera can agglutinate the baby's RBCs. This would not be useful for determining whether circulating IgG antibody to the Rh factor is present in the serum from the mother. An indirect Coombs test to measure IgM anti-Rh antibody (choice D) would not be informative, since IgM anti-Rh antibody will not cross the placenta. The Ouchterlony test, or double-diffusion precipitin reaction (choice E), is used to determine if two antigens are identical, unrelated, or related but not identical. An IgG2 molecule is composed of which of the following? A. One alpha, one gamma2, and two kappa chains B. One gamma1 chain and two kappa chains C. Two gamma1 chains and one kappa and one lambda chain D. Two gamma1 chains and two kappa chains E. Two gamma2 chains and two kappa chains

Explanation:

The correct answer is E. IgG molecules contain two gamma heavy chains of a given subtype and two light chains (either kappa or lambda). The 2 in IgG2 indicates the subclass to which the molecule belongs. IgG2 contains two gamma2 chains (since a given B cell can only form one type of heavy chain). The IgG molecule will contain either two kappa chains or two lambda chains, but never one of each (choice C). A given cell produces immunoglobulin molecules with a single type of heavy chain (compare with choice A). IgG molecules with gamma1 chains (choices B, C, and D) would be of the IgG1 subclass.

A 42-year-old Nigerian in the United States visiting his brother comes to the clinic with complaints of several months of weight loss, night sweats, sputum production, and occasional spitting up blood. His physician places a tuberculin skin test and the results are positive. This positive tuberculin test indicates which of the following?

A. A cell-mediated immune response has occurred
B. A humoral immune response to TB is positive
C. The B and T cell systems are functional
D. The B cell system is functional
E. The phagocytic neutrophilic system is functional

Explanation:

The correct answer is A. The tuberculin skin test is a delayed type hypersensitivity test. This type IV hypersensitivity is a cell-mediated immune response involving CD4 T helper 1 (TH1) lymphocytes. This individual is TB positive, and should now have cultures and a chest x-ray to confirm an active case of tuberculosis. Elimination of tuberculosis, like all intracellular bacterial infections, is not dependent on a humoral (antibody) response (choice B). This patient could have antibody to tuberculosis, but recovery is entirely dependent on cell-mediated immunity. The positive tuberculin test is a result of functioning CD4 TH1 cells, but does not depend on a functional B cell system (compare with choice C). The B cell is involved in the production of immunoglobulins. This disease is dependent on cell mediated (T cell immunity) and not B cell production of immunoglobulins (antibody) (compare with choice D). The positive tuberculin test does not involve phagocytic neutrophils (compare with choice E). The primary cell involved is the CD4 TH1 lymphocyte.

A 6-year-old boy with a sore throat, cough, and fever is noted to have a marked elevation in his serum C-reactive protein. This finding indicates which of the following?

A. Developing autoimmune reaction
B. Ineffective immune response
C. Meningitis
D. Non-specific inflammation
E. Respiratory compromise

Explanation:

The correct answer is D. C-reactive protein is one of the most commonly measured acute-phase reactants, which are a group of serum proteins showing a rapid increase in serum concentration in response to any inflammatory process. This finding is entirely non-specific–it only indicates a recent inflammatory process. An autoimmune reaction (choice A), which is certainly a concern with streptococcal pharyngitis, is suggested by the development of a rising ASO (antistreptolysin O) titer weeks after the illness. Acute phase reactants are not specific to autoimmune processes. Increases in C-reactive protein indicate a healthy immune response to an infective pathogen. An ineffective immune response (choice B) would not elicit acute-phase reactions. Meningitis (choice C) cannot be diagnosed from a blood test. The diagnosis of meningitis requires the appropriate clinical signs (neck stiffness, mental status changes) and a positive lumbar puncture.
Respiratory compromise (choice E) produces changes in arterial blood gases and blood pH. Acute-phase reactants do not reflect respiratory status.

Which of the following substances takes part in the nonoxidative killing pathway by which pathogenic bacteria can be killed?

A. Hydrogen peroxide
B. Hypochlorous acid
C. Lysozyme
D. Myeloperoxidase
E. Superoxide ions

Explanation:

The correct answer is C. Lysozyme is present in tears, saliva, mucus, vaginal secretions, and several other body fluids. This material lyses the peptidoglycan layer of the cell wall of bacteria without participation of any of the elements of the oxidative killing pathway. Hydrogen peroxide (choice A) is formed in the oxidative killing pathway from the superoxide ions. This material is also very toxic to bacteria pathogens. Hypochlorous acid (choice B) is formed from hydrogen peroxide and halide ions in the presence of myeloperoxidase. Myeloperoxidase (choice D) is an enzyme present in the oxidative killing pathway that breaks down hydrogen peroxide. Superoxide ions (choice E) are formed by NADPH oxidase in the first reaction of the oxidative killing pathway. These superoxide ions are extremely toxic to bacterial organisms.

An alert pediatric intern notices that a neonate with dysmorphic facies is twitching abnormally. As he watches, the baby experiences a seizure. Stat laboratories indicate a glucose of 90, serum sodium of 140, serum potassium of 4.2 and serum calcium of 3.9. Over the next several months, the child is admitted to the hospital twice for Candida infections, and once for a viral exanthem. Which of the following is the most likely diagnosis?

A. Ataxia telangiectasia
B. Bruton's hypogammaglobulinemia
C. DiGeorge syndrome
D. Severe combined immunodeficiency
E. Wiskott-Aldrich syndrome

Explanation:

The correct answer is C. DiGeorge syndrome is due to fetal malformation of epithelial elements of the third and fourth pharyngeal pouches, leading to thymic aplasia or severe hypoplasia and sometimes, absence of the parathyroids. Clinically, the condition is classified as a selective T-cell deficiency, but varying degrees of deficiency of antibody production may also be seen. Affected infants are susceptible to fungal and viral infections, and may exhibit tetany due to severe hypocalcemia. In autosomal recessive ataxia telangiectasia (choice A), progressive cerebellar ataxia is accompanied by multiple telangiectasias on exposed skin and a variable, usually moderate, immunodeficiency that may involve both antibody production and cell-mediated immunity. Chronic or recurrent sinus and pulmonary infections result in bronchiectasis. In Bruton's (X-linked) agammaglobulinemia (choice B), children have normal numbers of circulating T cells with very few circulating B cells and only tiny amounts of circulating antibody. Affected individuals have recurrent bacterial infections beginning late in the first year of life. Severe combined immunodeficiency (choice D) is actually a cluster of several diseases with variable genetics characterized by severely deficient T cell functions and variable (often depressed) antibody production. Multiple, sometimes simultaneous, infections with viruses, bacteria, and fungi occur. Wiskott-Aldrich syndrome (choice E) is an X-linked disease with thrombocytopenia, lymphopenia, and decreased T cell function. Lymphoid malignancy (e.g., acute lymphocytic leukemia) may occur.

A 38-year-old woman with systemic lupus erythematosus (SLE) is seen by her family physician. On her last visit, he sent blood to the laboratory for an antinuclear antibody panel. Based on the results, he recommends that the patient be seen by a nephrologist. A high titer of autoantibodies directed against which of the following antigens most likely prompted his decision?

A. Double-stranded DNA (dsDNA)
B. Histones
C. Ribonucleoprotein (RNP)
D. Smith antigen (Sm)
E. Single-stranded DNA (ssDNA)

Explanation:

The correct answer is A. Antibodies to double-stranded DNA, unlike most other autoantibodies, are considered to have prognostic significance in SLE. High titers of this antibody are associated with the development or progression of lupus nephritis. Lupus nephritis can occur in the absence of any demonstrable abnormalities on urinalysis; renal biopsy is helpful in such cases to document disease progression.
Anti-histone antibodies (choice B) can be seen as one of several autoantibodies in SLE or as an isolated finding in drug-induced lupus.
High titers of anti-RNP (choice C) specifically suggests mixed connective tissue disease rather than lupus,
although 30-40% of lupus patients will have this antibody.
Anti-Sm (choice D) is specific for SLE, but does not predict outcome.
Anti-ssDNA autoantibodies (choice E) are a nonspecific feature of SLE and can be seen in other autoimmune diseases.

A 28-year-old mother gives birth to her first child. The father is homozygous Rh D positive and the mother is homozygous Rh D negative. The baby is born without any complications, but the mother is not given RhoGAM (anti-Rh IgG) following the delivery. Eighteen months later she delivers another child, who is anemic, slightly jaundiced, and has an enlarged spleen and liver. Which type of hypersensitivity best describes this condition?

A. Atopic disease
B. Cytotoxic disease
C. Delayed hypersensitivity
D. Immediate hypersensitivity
E. Immune complex disease

Explanation:

The correct answer is B. This is an example of type II hypersensitivity, in which antibody is produced against cells or receptors in the body. In this type of cytotoxic disease, the mother produced antibody against the Rh D positive cells to which she was exposed upon delivery of her first child. This child must be Rh D positive, because the father is homozygous Rh D positive. The Rh D cells are foreign to the mother since she is Rh D negative, and so she mounted an antibody response against the antigen. The second child also must have been Rh D positive, and the mother's antibody crossed the placenta resulting in the disease. Atopic disease (choice A) is a Type I immediate hypersensitivity reaction that occurs in minutes following a second exposure to the offending allergen. The antigen could be ragweed, tree pollen, grass pollen, animal fur, or food products. Type IV hypersensitivity is delayed type hypersensitivity (choice C), developing in response to viral, fungal, and intracellular bacterial antigens. It involves CD4 T helper lymphocytes, but not antibody. Immediate hypersensitivity (choice D) is a Type I reaction that occurs in minutes following a second exposure to the offending allergen. Common antigens include ragweed, tree pollen, grass pollen, animal fur, or food products. Immune complex disease (choice E) is a Type III hypersensitivity reaction. It involves antibody and complement, but in the form of antigen-antibody-complement complexes. Rheumatoid arthritis, systemic lupus erythematous, and serum sickness are examples of type III hypersensitivity.

A 21-year-old man presents with cough, fever, and hemoptysis. Blood tests show significantly elevated BUN and creatinine. Immunofluorescent microscopy reveals a diffuse linear pattern of fluorescence along the basement membranes of alveolar septa and glomerular capillaries. Which type of hypersensitivity is associated with this disease?

A. Type I
B. Type II
C. Type III
D. Type IV

Explanation:
The correct answer is B. This patient has a USMLE-favorite disease: Goodpasture's syndrome, which affects both the renal and pulmonary systems. In the kidney, it causes a rapidly progressive glomerulonephritis caused by antibodies directed against a collagen component of the glomerular basement membrane (anti-GBM antibody––a classic clue to this diagnosis). These antibodies create a linear pattern on immunofluorescence. Note that they are also active against the basement membrane of respiratory alveoli, accounting for the pulmonary component of the disease. Autoimmune reactions such as those found in Goodpasture's, certain drug allergies, blood transfusion reactions, and hemolytic disease of the newborn are classified as Type II hypersensitivities (antibody-mediated cytotoxicity). IgG or IgM antibody reacts with membrane-associated antigen on the surface of cells, causing activation of the complement cascade and, ultimately, cell destruction. Type I (choice A) (immediate, atopic, or anaphylactic) reactions require an initial (sensitizing) exposure to an antigen. Upon re-exposure to the antigen, cross-linking of IgE receptors occurs on the surface of basophils and mast cells. The mast cells then release a variety of mediators, including histamine. Clinical syndromes include asthma, atopic dermatitis, eczema, and allergic rhinitis. Hives ("wheal and flare") are characteristic of Type I hypersensitivity. Type III (choice C) hypersensitivity (immune complex-mediated hypersensitivity) is caused by antibodies to foreign antigens. Immune complexes of IgG or IgM with the antigen activate complement, resulting in the generation of C3b, which promotes neutrophil adherence to blood vessel walls. The complexes also generate C3a and C5a (anaphylatoxins), which lead to inflammation and tissue destruction. The hallmark signs of Type III sickness occur 7-14 days after exposure to the offending antigen and include urticaria, angioedema, fever, chills, malaise, and glomerulonephritis. Clinical syndromes include serum sickness (e.g., penicillin, streptomycin, sulfonamide, phenylbutazone hypersensitivity) and the Arthus response. Immune complexes are also observed in lupus (SLE). Type III glomerulonephritis (e.g., poststreptococcal glomerulonephritis) is characterized by a "lumpy bumpy" appearance on immunofluorescence using tagged antibody specific for immunoglobulin or complement.

Type IV (choice D) is also known as delayed-type hypersensitivity (DTH). Unlike the other types, which are mediated by antibody, DTH depends on TDTH cells that have been sensitized to a particular antigen. T cells react with antigen in association with MHC class I gene products and release lymphokines. Examples include tuberculin skin sensitivity and contact dermatitis (e.g., poison ivy rash).

Which of the following enzymes does the neutrophil use to initiate the production of toxic oxygen compounds that kill bacteria?

A. Hydrogen peroxide
B. Myeloperoxidase
C. NADPH oxidase
D. Superoxide
E. Superoxide dismutase

Explanation: The correct answer is C. The first step in killing bacteria is the production of superoxide ion, O2- , by the action of NADPH oxidase on NADPH and O2. The superoxide is then converted to hydrogen peroxide, either spontaneously or through the action of superoxide dismutase. The hydrogen peroxide can also be converted to the toxic HOCl. radical by the action of myeloperoxidase.

Hydrogen peroxide(choice A) is not an enzyme.

Myeloperoxidase (choice B) is part of the killing pathway, but is active later in the sequence, when H2O2 is

converted in the presence of Cl- to the HOCl. radical.

Superoxide (choice D) is not an enzyme.

Superoxide dismutase (choice E) is part of the killing pathway, but is active later in the sequence, when

superoxide is converted to hydrogen peroxide.

Liver biopsy in a patient with cirrhosis demonstrates markedly increased iron stores. The patient has no known

history of hemolytic anemia or other potential causes of a secondary increase in hepatic iron stores. This

patient's primary disorder is most strongly associated with which of the following HLA alleles?

A. HLA-A3
B. HLA-B27
C. HLA-DR2
D. HLA-DR3
E. HLA-DR4

Explanation:

The correct answer is A. The disease is primary hemochromatosis, in which abnormal iron accumulation occurs in many different organs. The liver may become cirrhotic, and the patient may become diabetic due to pancreatic damage. Heart failure may occur due to myocardial dysfunction, and the skin can take on a bronze

discoloration. In fact, the old term for hemochromatosis was "bronze diabetes." Primary hemochromatosis is associated with the HLA-A3 allele.

HLA-B27 (choice B) is associated with psoriasis, ankylosing spondylitis, inflammatory bowel disease, and Reiter's syndrome.

HLA-DR2 (choice C), together with HLA-DR3 (choice D), is associated with systemic lupus erythematosus.

HLA-DR3 (choice D) is additionally associated with Sjögren's syndrome, chronic active hepatitis, and type 1 diabetes mellitus (with HLA-DR4, choice E).

In addition to being associated with type 1 diabetes, HLA-DR4 (choice E) is also associated with rheumatoid arthritis.

* What should be included in the initial immune evaluation for a patient that has suffered unexplained recurrent miscarriage?
* What should be included in the initial immune evaluation for a patient that is contemplating ivf or that has suffered failed ivf?
* When should laboratory tests be done?
* Who can order an immune evaluation?
* Who will interpret the laboratory results?
* If an immune problem is identified, when should medication be started and how long would it be continued?
* How does ivig work?
* Why does RIA prefer prednisone to other forms of corticosteroid like dexamethasone?
* If a woman has had at least one successful pregnancy without immune treatment, does this mean that immune issues can be excluded?
* What is the cost of an immune evaluation for a patient with recurrent miscarriage?
* What is the cost of an imune evaluation for a patient contemplating ivf or who has suffered failed ivf?
* Does reproductive immunology associates participate with any insurance company?
* Does health insurance pay for the laboratory tests and treatment?
* Is it necessary to go to Los Angeles to become a patient and/or receive treatment?
* How can I send my blood to RIA to be tested?

Q: What should be included in the initial immune evaluation for a patient that hassuffered unexplained recurrent miscarriage?

The workup should include the following tests:

* Leukocyte antibody detection/blocking antibodies.
* Antiphospholipid antibody panel (7 epitopes, 3 isotypes).
* Antinuclear antibody panel.
* Immunophenotype for NK cell number.
* NK/IVIg assay for NK cell activity.

Also very useful:

* TNF alpha (intracellular) assay. * Gene mutations for inherited thrombophilias. * DQ alpha genotyping (couple).

Q: What should be included in the initial immune evaluation for a patient that is contemplating ivf or that has suffered failed ivf?

The workup should include the following tests:

* Antiphospholipid antibody panel (7 epitopes, 3 isotypes).
* Antinuclear antibody panel.
* Antithyroid antibody panel.
* Immunophenotype for NK cell number.
* NK/IVIg assay for NK cell activity.

Also very useful:

* TNF alpha (intracellular) assay.
* Gene mutations for inherited thrombophilias.
* DQ alpha genotyping (couple).
* Leukocyte antibody detection/blocking antibodies.

Q: When should laboratory tests be done?

Please see our page When to Test. Q: Who can order an immune evaluation?

A physician or laboratory, hospital or clinic on behalf of the physician must order the laboratory tests. Q: Who will interpret the laboratory results?

The ordering physician must interpret laboratory tests. Liability issues discourage a RIA physician from interpreting tests and/or making specific treatment recommendations for a patient that is not directly under our care. A RIA physician is always available to other physicians to discuss laboratory results and make recommendations.

Laboratories are prohibited from providing test results (verbal, mail or facsimile) to patients to avoid self-diagnosis and treatment. Q: If an immune problem is identified, when should medication be started and how long would it be continued?

All medication must be started prior to conception for maximal benefit.

For patients having natural conception or IUI, heparin, aspirin and prednisone are started 5-7 days prior to ovulation, and IVIg is administered 7-10 days prior to ovulation.

For patients undergoing IVF, prednisone is usually started 4 weeks before IVF transfer, heparin and aspirin started 2 weeks before IVF transfer, and IVIg is administered 7-10 days prior to transfer. Q: How does ivig work?

IVIg works in several ways. For patients who lack blocking antibodies, IVIg provides temporary, non-specific blocking antibodies that will circulate in the mother's body for about one month to protect the fetus. IVIg also down regulates NK cell activity so that they do not damage the fetus. The mechanism by which this happens is very complex; antibodies in the IVIg can attach onto NK cells, signaling them to decrease their killing activity. The effect on the NK cells lasts about one month at which time retesting is recommended. Q: Why does RIA prefer prednisone to other forms of corticosteroid like dexamethasone?

Prednisone is the preferred corticosteroid for several reasons. First, it is highly bound to the blood protein albumin. This complex is large and has difficulty crossing the placenta. In addition, an enzyme in the placenta, beta-2-dehydrogenase, inactivates any free prednisone that does get across the placenta. The fetal liver is unable to reactivate the prednisone until about week 30 of gestation. At that time, the active steroid may help increase surfactant production, which helps to mature the fetal lungs. A patient would have to take more than 60 mg of prednisone per day to affect the developing baby.

Q: If a woman has had at least one successful pregnancy without immune treatment, does this mean that immune issues can be excluded?

Patients who fall into this category are called secondary recurrent spontaneous aborters. Immune problems can develop after a woman has had a normal pregnancy. When this occurs, it becomes very difficult for her to carry a fetus to term without treatment. Q: What is the cost of an immune evaluation for a patient with recurrent miscarriage?

The cost of a comprehensive immune workup for recurrent miscarriage is approximately $1300. Some of the tests need to be repeated periodically to adjust medication doses or to identify evolving immune abnormalities.

Genetic studies are an additional charge. Fortunately, these do not have to be repeated because they never change.

Q: What is the cost of an imune evaluation for a patient contemplating ivf or who has suffered failed ivf?

The cost of a comprehensive immune workup for failed IVF is approximately $1100. Some of the tests need to be repeated periodically to adjust medication doses and to identify evolving immune abnormalities.

Genetic studies are an additional charge. Fortunately, these do not have to be repeated because they never change.

Q: Does reproductive immunology associates participate with any insurance company?

Reproductive Immunology Associates does not participate with any insurance company. Payment must accompany a laboratory specimen, unless, your physician has an account with our laboratory. We accept most major credit cards, and personal, bank or cashier's checks, or money orders.

If you include a copy of your insurance card (front and back), we will submit a health claim form to your insurance company, on your behalf. You will be required to download the registration form (LINK), complete all of the information on the first page, and return it to us along with the copy of your insurance card. A letter will accompany the health claim form instructing the insurance company to reimburse the patient directly. Q: Does health insurance pay for the laboratory tests and treatment?

Health insurance companies vary significantly in their willingness to pay for matters related to reproduction. Since 1990, the number of insurance companies that are paying has steadily increased. Some insurance companies pay for all procedures and treatments less the deductible and co-pay. Much of the success at getting re-imbursement depends upon diagnostic coding. For patients that are directly under the care of a RIA physician, we will provide letters of medical necessity and journal references to support our position. These measures do not guarantee success at re-imbursement; however, they have worked for a number of patients. Q: Is it necessary to go to Los Angeles to become a patient and/or receive treatment?

Telephone consultations are available to patients outside of the Southern California area. A patient that chooses to be treated by a RIA physician will have to have the initial treatment on site. She will also need a physician in her geographic location for routine and semi-urgent evaluation and treatment. If the patient has a local physician who is willing to accept recommendations and work with a RIA physician, a trip to Los Angeles may not be necessary.

Q: How can I send my blood to RIA to be tested?

Fortunately, blood is easily shipped via express type mail, for example, FEDEX, DHL, AIRBORNE, UPS priority.

For international patients, the specimen would have to go through customs. As part of the declaration, you would have to say that "the blood is non-infectious and for diagnostic testing".

1. Which statement about hypersensitivity is correct?
a. Type III and type II hypersensitivity are similar, but differ in that type III reactions involve "self" antigens and type II reactions do not.
b. Mediators of type IV hypersensitivity include IFN-gamma as well as factors that affect macrophage chemotaxis or migration.

c. Histamine and leukotrienes, both released by mast cells, are mediators of inflammation, but not hypersensitivity reactions.

2. Which statement related to vaccines and vaccination is correct?
a. The main strategy in immunization against viral diseases is to generate as many memory B cells as possible, so responses to infection will be quicker.
b. Current vaccines used to prevent diseases caused by toxins generally contain chemically-inactivated toxin, referred to as toxoid.
c. Research has shown that plants genetically-engineered to contain antigens induce tolerance rather than stimulating the immune system when ingested.

MATCHING MULTIPLE CHOICE: Choose the BEST answer. An answer may be used more than once within a set of questions.

Answers to questions 3-4:
a. CTLs
b. Macrophages
c. Eosinophils
d. NK cells
e. More than one of these

3. Immunity against the larval forms of certain intestinal parasites can involve the participation of _____, and these cells also accumulate at sites where atopic hypersensitivity reactions are occurring so this effect is often used by pathologists as indicators of this type of hypersensitivity reaction.

4. _____ participate in antibody-dependent cellular cytotoxicity (ADCC) by virtue of having FcR and granules that contain factors that can lyse target cells.

TRUE-FALSE: If a statement is TRUE, enter A in the blank; if it is FALSE, enter B.

5. Arthus reactions are typified by formation of bulky immune complexes in the kidneys, lungs, and joints, whereas serum sickness is typified by formation of soluble immune complexes at a local site of administration of the eliciting dose of antigen.

6. The hemostasis observed in acute inflammatory responses results from constriction of the arteries and dilation of the veins in the immediate area, and results in the exudation of fluid, but not cells, into an area where cell damage has occurred.

Answering these essay questions will lead you to a better understanding of the concepts we covered during the fourth section of this course.

The mythical Gram-negative extracellular bacterium, Stevensonella pulmonensis, causes a disease called acute miamitis as the result of its thick capsule and production of a toxin that causes buildup of fluid in the lungs. Which immune mechanism(s) would be effective against this bacterium and why?

* Antibody specific for the toxin would be helpful because it might neutralize the toxin by preventing its interaction with cellular targets.
* Activated macrophages would be required because these cells would be able to prevent intracellular multiplication of this pathogen.
* Opsonizing antibody specific for capsular antigens might be helpful because it would be able to enhance phagocytosis of the bacterium by neutrophils, which could then kill it intracellularly.
* ADCC mediated by NK cells would be a primary line of defense against this pathogen because it would provide for its complement mediated lysis.

Define each of these terms, then briefly explain the relationships among them: Lymphocyte - Natural killer (NK) cell - Cytotoxic T lymphocyte (CTL) -

Generate a single overview diagram that shows how Th1 and Th2 cells interact with APCs, B cells, Tc cells, and macrophages to assist in generation of antibody responses, CTL responses and activated macrophage responses, respectively. (Note: Be certain to label each of the cells, indicate the cytokines involved, and provide brief descriptions of the processes that occur as a result of each of the interactions.)

Generate a diagram that depicts the events in T cell signaling (both signal 1 and signal 2) then compare and contrast (in words) this pathway with the events that occur in B cell signaling (both signal 1 and signal 2).

Generate a diagram that depicts the events in T cell development in the thymus, then compare and contrast (in words) this pathway with the events that occur in B cell development in the bone marrow.
Describe the process of apoptosis, and then explain how it is induced and carried out in each of these instances:
a. Positive and negative selection in either T or B cell development
b. Regulation of T cell responses
c. NK cell and CTL function
d. Eosinophil function
Discuss (at the mechanistic level) the role of antigen processing and presentation in determining the outcome of an immune response against each of these:
a. Extracellular parasite
b. Obligate intracellular parasite
c. Facultative intracellular parasite.
(To answer this question, you will need to choose a specific microbe that is applicable to each of these situations, and you will need to explain, in detail, how the antigen processing pathways used in each instance influence the response.)

Discuss, in general, how receptors function vis-Ã -vis binding of ligand and initiation of intracellular signaling, then explain how this works for each of these:
a. IFN-gamma receptor
b. TCR
c. BCR and B cell coreceptor

Describe, in detail, the mechanisms involved in generating TCR diversity, then compare and contrast these mechanisms with those involved in generating: a. BCR diversity
b. MHC diversity

Discuss the relationships between immunoglobulin structures of all five classes (IgG, IgM, IgA, IgD, IgE) of antibody and the functions that each of these types of antibody can carry out in antibody mediated immunity, including complement interactions.

Discuss the mechanisms or activation and regulation of the three complement activation pathways we discussed in class, then explain why each of the negative regulatory processes is necessary to prevent hypersensitivity and/or autoimmunity reactions. Draw a series of four diagrams depicting the events involved in margination and diapedesis of PMNs. In your diagrams, be sure to include cell surface molecule interactions, chemokines that influence these interactions, and the process of diapedesis. Also, be sure to label every component and to include short descriptions of what is occurring in each diagram. Finally, compare and contrast this series of events with those that occur during emigration of lymphocytes from the bloodstream into lymphoid tissue.

Generate a series of four diagrams (one for each major step) depicting the sequence of interactions involved in activation of a B cell by a Th cell, starting with binding antigen to BCRs, and finishing with activation of the B cell by cytokines secreted by the Th cell. Be certain that your diagrams are clearly labeled, and complete your answer by adding a text description that outlines the mechanisms involved in each step.

1. Why do organisms require "microscopic" defense mechanisms? What is there to defend against? Why do organisms have internal defense mechanisms? What is there to defend against inside us?

2. What is an antibody? - What are antibodies used for? - How are they produced?

3. How are "natural killer cells" and "cytotoxic T cells" similar; how are they fundamentally different?

4. Cells involved in the immune response are white blood cells. What are the different kinds? Where are they made?

5. The "third line of defense" includes several kinds of B-cells, T-cells, macrophages, etc. organized in (1) humoral immune response and (2) cell-mediated immune response. These two responses relate to (1) humoral response: stimulus can be a small "free" molecule (antigen) floating around in the blood vs. (2) cell-mediated response: stimulus can ONLY be a "foreign" molecule (antigen) attached to the membrane of another cell (antigen presenting cell). B-cells are part of the humoral response pathway, capable of responding to "free" antigens. T-cells are part of the cell-mediated pathway, capable ONLY of responding to antigen presenting cells.

6. T-cells have evolved to respond to our own cells when they have either: (1) become infected by some virus, or (2) become "strangers" (i.e. loose the molecular identity signaling relation to self).

7. Regarding the humoral immune response pathway, be familiar with the following: antigen, antibody, immunoglobulin, clonal selection, B-cell, plasma antigen producing cells, memory B-cells.

8. What is the basic structure of IgG antibody? What are the ways in which an IgG antibody can inactivate antigens?

9. Regarding the cell-mediated immune response pathway, be familiar with the following: antigen, macrophage, antigen presenting cell, helper T-cell (Th-cell), cytotoxic T-cell (Tc-cell), Memory Tc-cells, Memory Th-cells.

10. Specificity of the "third line of defense" involves receptors. B-cell receptors are proteins, and therefore gene products, with nearly identical structure to the IgG molecules that B-cell progeny will later secrete. Following activation by a specific antigen and subsequent clonal selection, B-cell progeny produce free IgG antibodies with nearly the same structure as their receptors. There are more than 1,000,000 different B-cell receptors, recognizing an equivalent number of different potential antigens. However, there are only about 100,000 different genes in us. How then, is such diversity of B-cell receptor generated?

11. T-cells also have receptors, like B-cells and with similar diversity. A difference between B-cells and T-cells is that B-cell progeny produce "free" antibodies while activated T-cells either produce hormones (interleukins) which activate other responses or produce cytotoxic progeny which kill antigen presenting cells. T-cells do not produce "free" antibodies.

12. What are antibodies used for?

- How are they produced?
13. What is MHC? How is MHC involved in identifying self? If humans have 20 MHC genes and there are 100 different alleles for each gene, how many different human identities are possible?

14. What is "complement"? Where does it come from? How does it act? Here are further guidelines.